Obstetrics COPY Flashcards

1
Q

Define:

  1. Embryo
  2. Fetus
  3. Infant
A
  1. Embryo: fertilization to eight weeks
  2. Fetus: eight weeks to birth
  3. Infant: birth to one year old

Fischer, Conrad (2012-11-01). Master the Boards USMLE Step 2 CK (Page 441). Kaplan Medical Test Prep. Kindle Edition.

How well did you know this?
1
Not at all
2
3
4
5
Perfectly
2
Q

A 27-year-old woman presents with nausea and vomiting for the past 2 weeks. Symptoms are worse in the morning, but can occur at any time during the day.
She has a decrease in appetite. Her last menstrual period (LMP) was 6 weeks ago. Physical examination is unremarkable. Which of the following is the best next step in the management of this patient?
a. Complete blood count
b. Beta-HCG
c. HIDA scan
d. Comprehensive metabolic panel
e. Urinanaylsis

Fischer, Conrad (2012-11-01). Master the Boards USMLE Step 2 CK (Page 441). Kaplan Medical Test Prep. Kindle Edition.

A

B. A pregnancy test should be done first in all symptomatic women of childbearing age. Her LMP occurred 6 weeks ago and the patient is experiencing
“morning sickness.” Morning sickness is caused by an increase in beta-HCG produced by the placenta. This can occur until the 12th to 14th week of pregnancy.
A complete blood count (CBC), comprehensive metabolic panel (CMP), and urinalysis are used to evaluate the severity of dehydration, not the etiology. A HIDA scan is done in patients with suspected cholecystitis.

Fischer, Conrad (2012-11-01). Master the Boards USMLE Step 2 CK (Page 441). Kaplan Medical Test Prep. Kindle Edition.

How well did you know this?
1
Not at all
2
3
4
5
Perfectly
3
Q

Dating Methods:

  1. Define developmental age (DA)
  2. Define gestational age (GA)
  3. What is Nägele rule
  4. What is the estimated date of delivery in a woman with an LMP of July 1, 2010?
A
  1. Number of days since fertilization
  2. Number of days/weeks since the last menstrual period (usually 2 weeks longer than DA)
  3. Estimation of the day of delivery by taking the last menstrualperiod, subtracting 3 months, and adding 7 days. LMP - 3 months + 7 days
  4. April 8, 2011

Fischer, Conrad (2012-11-01). Master the Boards USMLE Step 2 CK (Page 442). Kaplan Medical Test Prep. Kindle Edition.

How well did you know this?
1
Not at all
2
3
4
5
Perfectly
4
Q

Define:

  1. First trimester
  2. Second trimester
  3. Third trimester
A
  1. Fertilization until 12 weeks (DA) or 14 weeks (GA)
  2. 12(DA)/14(GA) weeks until the 24 week (DA) or 26 week (GA)
  3. 24(DA)/26(GA) weeks until delivery

Fischer, Conrad (2012-11-01). Master the Boards USMLE Step 2 CK (Page 442). Kaplan Medical Test Prep. Kindle Edition.

How well did you know this?
1
Not at all
2
3
4
5
Perfectly
5
Q

When is the fetus deifned as

  1. Pre-viable
  2. Preterm
  3. Term
  4. Postterm
A
  1. Fetus born before 24 weeks
  2. Fetus born between 25 and 37 weeks
  3. Fetus born between 38 and 42 weeks
  4. Fetus born after 42 weeks

Fischer, Conrad (2012-11-01). Master the Boards USMLE Step 2 CK (Page 442). Kaplan Medical Test Prep. Kindle Edition.

How well did you know this?
1
Not at all
2
3
4
5
Perfectly
6
Q

Gravidity is the number of times a patient has been pregnant. Parity is what happens to the pregnancy. This is broken down into 4 numbers:
1. Full-term births
2. Preterm births
3. Abortions (both spontaneous and induced)
4. Living children (if a patient has a multiple gestation pregnancy, one birth
results in 2 living children)

Write the conventional notation for a 35-year-old woman presents to the office for her sixth pregnancy. She has had 2 abortions, 2 children born at term, and a set of twins born preterm.

A

G6P2124

Use: Full-term birth (F); Preterm birth (P); Abortions (A); Living children (L) = F-PAL

Fischer, Conrad (2012-11-01). Master the Boards USMLE Step 2 CK (Page 442). Kaplan Medical Test Prep. Kindle Edition.

How well did you know this?
1
Not at all
2
3
4
5
Perfectly
7
Q

A 20-year-old woman presents to the office because she believes that she is pregnant. Her sexual partner usually pulls out, but did not do so 2 weeks ago. She is
now 4 weeks late for her menstruation. Which of the following is one of the first signs of pregnancy found on physical exam?
a. Quickening
b. Goodell sign
c. Ladin sign
d. Linea nigra
e. Chloasma

Fischer, Conrad (2012-11-01). Master the Boards USMLE Step 2 CK (Page 443). Kaplan Medical Test Prep. Kindle Edition.

A

B. One of the first signs of pregnancy that is seen on physical exam is the Goodell sign, softening of the cervix that is felt first at 4 weeks. Quickening is the first
time the mother feels fetal movement.

Fischer, Conrad (2012-11-01). Master the Boards USMLE Step 2 CK (Page 443). Kaplan Medical Test Prep. Kindle Edition.

How well did you know this?
1
Not at all
2
3
4
5
Perfectly
8
Q

Based on the following physical signs, list the sign (name of sign) and the time form conception when it is seen

  1. Softening of the cervix
  2. Softening of the midline of the uterus
  3. Blue discoloration of vagina and cervix
  4. Small blood vessels/reddening of the palms
  5. The “mask of pregnancy” is a hyperpigmentation of the face most commonly on forehead, nose, and cheeks; it can worsen with sun exposure
  6. A line of hyperpigmentation that can extend from xiphoid process to pubic symphysis

Fischer, Conrad (2012-11-01). Master the Boards USMLE Step 2 CK (Page 444). Kaplan Medical Test Prep. Kindle Edition.

A
  1. Goodell sign; 4 weeks (first trimester)
  2. Ladin sign; 6 weeks (first trimester)
  3. Chadwick sign; 6–8 weeks (first trimester)
  4. Telangiectasias/palmar erythema; First trimester
  5. Chloasma; 16 weeks (second trimester)
  6. Linea nigra; Second trimester

Fischer, Conrad (2012-11-01). Master the Boards USMLE Step 2 CK (Page 444). Kaplan Medical Test Prep. Kindle Edition.

How well did you know this?
1
Not at all
2
3
4
5
Perfectly
9
Q

First trimester prenatal care:

  1. How often is the pregnant patient seen?
  2. When is ultrasund used to confirm GA and check for nuchal translucency
  3. A thickened or enlarged nuchal translucency indicates what condition?
A
  1. Every 4-6 weeks
  2. Between 11 and 14 weeks
  3. Down syndrome
How well did you know this?
1
Not at all
2
3
4
5
Perfectly
10
Q

A 17-year-old woman presents for a routine prenatal checkup at 12 weeks. Which of the following is the most accurate method to establish gestational age?

a. Ultrasound
b. Beta-HCG
c. Pelvic exam
d. Fundal height
e. LMP

Fischer, Conrad (2012-11-01). Master the Boards USMLE Step 2 CK (Page 445). Kaplan Medical Test Prep. Kindle Edition.

A

A. Ultrasound is the most accurate way of establishing gestational age at 11 to 14 weeks. Beta-HCG is unreliable in confirming dates, as the levels can be increased in twins or decreased in early abortions. Pelvic exam and fundal height are not the most
accurate methods to confirm dates because they may change with multiple gestations. A patient’s account of LMP is often unreliable because histories are inaccurately remembered.

Fischer, Conrad (2012-11-01). Master the Boards USMLE Step 2 CK (Page 446). Kaplan Medical Test Prep. Kindle Edition.

How well did you know this?
1
Not at all
2
3
4
5
Perfectly
11
Q

Second trimester prenatal care:

  1. When is the “triple” or “quad” screen done
  2. True or False: auscultation for the fetal heart is done now
  3. When is quickening felt
  4. When is routine ultrasound for fetal malformation done?
A
  1. At 15 to 20 weeks
  2. True
  3. At 16 to 20 weeks
  4. At 18 to 20 weeks

Fischer, Conrad (2012-11-01). Master the Boards USMLE Step 2 CK (Page 446). Kaplan Medical Test Prep. Kindle Edition.

How well did you know this?
1
Not at all
2
3
4
5
Perfectly
12
Q

What does an increase in MSAFP may indicate?

A
  • A dating error
  • Neural tube defect
  • Abdominal wall defect

Fischer, Conrad (2012-11-01). Master the Boards USMLE Step 2 CK (Page 446). Kaplan Medical Test Prep. Kindle Edition.

How well did you know this?
1
Not at all
2
3
4
5
Perfectly
13
Q
  1. List the components of the “triple” screen
  2. List the components of the “quad” screen
A
  1. A triple screen includes:
    • Maternal serum alpha fetoprotein (MSAFP)
    • BetaHCG
    • Estriol.
  2. The quad screen adds inhibin A to the triple screen.

Fischer, Conrad (2012-11-01). Master the Boards USMLE Step 2 CK (Page 446). Kaplan Medical Test Prep. Kindle Edition.

How well did you know this?
1
Not at all
2
3
4
5
Perfectly
14
Q

Third trimester prenatal care:

  1. Frequency of visits in 3rd timester
  2. Frequency of visit after 36 weeks
  3. What to do next if patient has continued Braxton-Hicks contraction?
A
  1. Every 2 to 3 weeks until 36 weeks
  2. Every week
  3. Check the cervix to rule out preterm labor before 37 weeks

Fischer, Conrad (2012-11-01). Master the Boards USMLE Step 2 CK (Page 446). Kaplan Medical Test Prep. Kindle Edition.

How well did you know this?
1
Not at all
2
3
4
5
Perfectly
15
Q

List the third trimester tests and the appropriate actions conducted at the following GA:

  1. 27 weeks
  2. 24-28 weeks
  3. 36 weeks
A
  1. 27 weeks:
    • Complete blood count
    • If hemoglobin
  2. 24–28 weeks:
    • Glucose load If glucose >140 at one hour, perform oral glucose tolerance test
  3. 36 weeks:
    • Cervical cultures for Chlamydia and gonorrhea; Rectovaginal culture for group B Streptococcus
    • Treatment if positive; Prophylactic antibiotics

Fischer, Conrad (2012-11-01). Master the Boards USMLE Step 2 CK (Page 446). Kaplan Medical Test Prep. Kindle Edition.

How well did you know this?
1
Not at all
2
3
4
5
Perfectly
16
Q

Glucose tolerance test in pregnancy:

  1. Describe the glucose loading test
  2. Describe glucose tolerance test in pregnancy
A
  1. Fasting or nonfasting ingestion of 50g of glucose, and serum glucose check 1 hour later
  2. Fasting serum glucose, ingestion of 100g of glucose, serum glucose check at 1, 2, and 3 hours. Elevated glucose during any two of these tests is gestational diabetes
How well did you know this?
1
Not at all
2
3
4
5
Perfectly
17
Q

True or False:

Stool softeners should be given with iron supplementation. Why?

A

True

Iron causes or worsens constipation

How well did you know this?
1
Not at all
2
3
4
5
Perfectly
18
Q

Chorionic Villus Sampling:

  1. When is it done and for whom?
  2. What it show?
  3. How is it done?
A
  1. Done at 10 to 13 weeks in advanced maternal age or known genetic disease in parent
  2. Obtains fetal karyotype
  3. Catheter into intrauterine cavity to aspirate chorionic villi from placenta (can be done transabdominally or transvaginally)

Fischer, Conrad (2012-11-01). Master the Boards USMLE Step 2 CK (Page 447). Kaplan Medical Test Prep. Kindle Edition.

How well did you know this?
1
Not at all
2
3
4
5
Perfectly
19
Q

Amniocentesis:

  1. When is it done and for whom?
  2. What it show?
  3. How is it done?
A
  1. Done after 11 to 14 weeks for advanced maternal age or known genetic disease in parent
  2. Obtains fetal karyotype (advanced maternal age)
  3. Needle transabdominally into the amniotic sac and withdraw amniotic fluid

Fischer, Conrad (2012-11-01). Master the Boards USMLE Step 2 CK (Page 447). Kaplan Medical Test Prep. Kindle Edition.

How well did you know this?
1
Not at all
2
3
4
5
Perfectly
20
Q

Fetal Blood Sampling:

  1. When is it done and for whom?
  2. What it show?
  3. How is it done?
A
  1. Percutaneous umbilical blood sample
  2. Done in patients with Rh isoimmunization and when a fetal CBC is needed
  3. Needle transabdominally into the uterus to get blood from the umbilical cord

Fischer, Conrad (2012-11-01). Master the Boards USMLE Step 2 CK (Page 447). Kaplan Medical Test Prep. Kindle Edition.

How well did you know this?
1
Not at all
2
3
4
5
Perfectly
21
Q

A 29-year-old woman with a past medical history of chlamydia presents with left lower quadrant abdominal pain for the past eight hours. She also states that she
has some abnormal vaginal bleeding. Her LMP was 6 weeks ago. On physical exam the patient’s temperature is 99°F, heart rate is 100 bpm, blood pressure is 130/80
mm Hg, and respiratory rate is 13 per minute.
Which of the following is the most likely diagnosis?
a. Ectopic pregnancy
b. Menstrual cramps
c. Diverticulitis
d. Ovarian torsion
e. Ovarian cyst

Fischer, Conrad (2012-11-01). Master the Boards USMLE Step 2 CK (Page 447). Kaplan Medical Test Prep. Kindle Edition.

A

A. See the following section on ectopic pregnancy. Diverticulitis causes left lower quadrant abdominal pain and rectal bleeding, not vaginal bleeding. The age
range of the patients has almost no overlap between ectopic pregnancy and diverticulitis. Ovarian torsion and ovarian cysts do not cause vaginal bleeding. Menstrual cramps are not associated with an altered menstrual pattern.

Fischer, Conrad (2012-11-01). Master the Boards USMLE Step 2 CK (Page 448). Kaplan Medical Test Prep. Kindle Edition.

How well did you know this?
1
Not at all
2
3
4
5
Perfectly
22
Q

List the risk factors of ectopic pregnancy

A
  • Pelvic inflammatory disease (PID)
  • Intrauterine devices (IUD)
  • Previous ectopic pregnancies (strongest risk factor)

Fischer, Conrad (2012-11-01). Master the Boards USMLE Step 2 CK (Page 448). Kaplan Medical Test Prep. Kindle Edition.

How well did you know this?
1
Not at all
2
3
4
5
Perfectly
23
Q

List the diagnostic tests in ectopic pregnancy

A
  • Beta-HCG: done to confirm the presence of a pregnancy
  • Ultrasound: to locate the site of implantation of the ectopic pregnancy
  • Laparoscopy: invasive test and treatment to visualize the ectopic pregnancy

Fischer, Conrad (2012-11-01). Master the Boards USMLE Step 2 CK (Page 448). Kaplan Medical Test Prep. Kindle Edition.

How well did you know this?
1
Not at all
2
3
4
5
Perfectly
24
Q

Outline the management algorithm in ectopic pregnancy

A

Below is the management algorithm for ectopic pregnancy. Unstable patients (low BP, high HR) should be given fluids and sent to surgery immediately.

How well did you know this?
1
Not at all
2
3
4
5
Perfectly
25
Q

Diagramatically outline the medical Rx outline for ectopic pregnancy

A

Medical Rx outline for ectopic pregnancy

How well did you know this?
1
Not at all
2
3
4
5
Perfectly
26
Q

Methotrexate is the drug of choice fo the medical Rx of ectopic pregnancy. List the baseline tests to be done before starting Rx with methotrexate.

A
  • CBC to monitor for anemia
  • Blood type/screen
  • Transaminases to detect changes indicating hepatotoxicity from the medications (e.g., methotrexate)
  • Beta-HCG to assess for success of treatment via a decrease in beta-HCG

Fischer, Conrad (2012-11-01). Master the Boards USMLE Step 2 CK (Page 449). Kaplan Medical Test Prep. Kindle Edition.

How well did you know this?
1
Not at all
2
3
4
5
Perfectly
27
Q

For how long is the patient followed to observe for a 15% decrease in the patient’s beta-hCG

A

4 to 7 days

Fischer, Conrad (2012-11-01). Master the Boards USMLE Step 2 CK (Page 449). Kaplan Medical Test Prep. Kindle Edition.

How well did you know this?
1
Not at all
2
3
4
5
Perfectly
28
Q

Exclusion criteria for medical Rx of ectopic pregnancy

A

Exclusion Criteria for Methotrexate

  • Immunodeficiency: Avoid methotrexate, which is an immunosuppressive drug
  • Noncompliant patients: Who knows if they will follow up? Patients need to return for evaluation to know if the treatment worked and if they need a second dose or surgery
  • Liver disease: Hepatotoxicity is a serious side effect of methotrexate. Baseline liver disease increases the risk of subsequent toxicity.
  • Ectopic is 3.5 cm or larger: The larger the ectopic, the greater the risk of treatment failure with methotrexate
  • Fetal heartbeat auscultated: A pregnancy developed enough to have a heartbeat detectable by auscultation has an increased risk of failure with methotrexate.

Fischer, Conrad (2012-11-01). Master the Boards USMLE Step 2 CK (Page 449). Kaplan Medical Test Prep. Kindle Edition.

How well did you know this?
1
Not at all
2
3
4
5
Perfectly
29
Q

Ectopic Pregnancy:

  1. Surgery done to preserve the Fallopian tube
  2. Surgery that removes the Fallopian tube
  3. True or False: Mothers who are Rh negative should also receive anti-D Rh immunoglobulin
A
  1. Salpingostomy
  2. Salpingectomy
  3. True

Fischer, Conrad (2012-11-01). Master the Boards USMLE Step 2 CK (Page 449). Kaplan Medical Test Prep. Kindle Edition.

How well did you know this?
1
Not at all
2
3
4
5
Perfectly
30
Q

A 20-year-old woman presents to the emergency department for vaginal bleeding and lower abdominal pain for one day. She states that she is 15 weeks pregnant. Vital signs include temperature 99.0°F, heart rate 100 bpm, blood pressure 110/75 mm Hg, and respiratory rate 12 per minute. On pelvic exam, there is blood present in the vault. Ultrasound shows intrauterine bleeding, products of conception,
and a dilated cervix. Which of the following is the most likely diagnosis in this patient?
a. Complete abortion
b. Incomplete abortion
c. Inevitable abortion
d. Threatened abortion
e. Septic abortion

Fischer, Conrad (2012-11-01). Master the Boards USMLE Step 2 CK (Page 450). Kaplan Medical Test Prep. Kindle Edition.

A

C. An inevitable abortion is characterized by intrauterine bleeding with a dilated cervix. (See table “Types of Abortions” for explanation of other answer choices.)

Fischer, Conrad (2012-11-01). Master the Boards USMLE Step 2 CK (Page 450). Kaplan Medical Test Prep. Kindle Edition.

How well did you know this?
1
Not at all
2
3
4
5
Perfectly
31
Q

Abortion is defined as a pregnancy that ends before 20 weeks gestation or a fetus less than 500 grams. Almost 80% of spontaneous abortions occur prior to 12 weeks gestation. Chromosomal abnormalities in the fetus account for 60% to 80% of spontaneous abortions. However, certain maternal factors increase the risk of abortion. List them.

A
  • Anatomic abnormalities
  • Infections (STDs)
  • Immunological factors (antiphospholipid syndrome)
  • Endocrinological factors (uncontrolled hyperthyroidism or diabetes)
  • Malnutrition
  • Trauma
  • Rh isoimmunization

Fischer, Conrad (2012-11-01). Master the Boards USMLE Step 2 CK (Page 450). Kaplan Medical Test Prep. Kindle Edition.

How well did you know this?
1
Not at all
2
3
4
5
Perfectly
32
Q

List the diagnostic tests in the management of abortion and the rationale for such tests

A
  • CBC to evaluate blood loss and need for transfusion
  • Blood type and Rh screen: should blood need to be transfused, and evaluation of need for anti-D Rh immunoglobulin
  • Ultrasound to distinguish between the types of abortions

Fischer, Conrad (2012-11-01). Master the Boards USMLE Step 2 CK (Page 451). Kaplan Medical Test Prep. Kindle Edition.

How well did you know this?
1
Not at all
2
3
4
5
Perfectly
33
Q

Based on the following ultrasound findings state the diagnosis and Rx for the following abortion presentations:

  1. No products of conception found
  2. Some products of conception found
  3. Products of conception intact,
    but intrauterine bleeding
    present and dilation of cervix
  4. Products of conception intact, intrauterine bleeding, no dilation of cervix
  5. Death of fetus, but all products of conception present in the uterus
  6. Infection of the uterus and the
    surrounding areas
A
  1. Complete abortion; Follow up in office
  2. Incomplete abortion; Dilation and curettage (D&C)/medical
  3. Inevitable abortion; D&C/medical
  4. Threatened abortion; Bed rest, pelvic rest
  5. Missed abortion; D&C/medical
  6. Septic abortion; D&C and IV antibiotics,such as levofloxacin and metronidazole

Fischer, Conrad (2012-11-01). Master the Boards USMLE Step 2 CK (Page 451). Kaplan Medical Test Prep. Kindle Edition.

How well did you know this?
1
Not at all
2
3
4
5
Perfectly
34
Q
  1. What agents are used for the medical Rx of abortion?
  2. True or False: Mothers who are Rh negative should also receive anti-D Rh immunoglobulin at this time.
A
  1. Misoprostol (a prostaglandin E1analog). These agents help open the cervix and expulse the fetus
  2. True

Fischer, Conrad (2012-11-01). Master the Boards USMLE Step 2 CK (Page 451). Kaplan Medical Test Prep. Kindle Edition.

How well did you know this?
1
Not at all
2
3
4
5
Perfectly
35
Q

List the complications of multiple gestation

A
  • Spontaneous abortion of one fetus
  • Premature labor and delivery
  • Placenta previa
  • Anemia

Fischer, Conrad (2012-11-01). Master the Boards USMLE Step 2 CK (Page 452). Kaplan Medical Test Prep. Kindle Edition.

How well did you know this?
1
Not at all
2
3
4
5
Perfectly
36
Q

A 28-year-old woman in her 28th week of pregnancy presents for severe lower back pain. She complains that the pain is cyclical and that it seems to be increasing in intensity. On physical examination, she seems to be in pain. Her temperature is 98.9°F, HR 104 bpm, BP 135/80 mm Hg, RR 15 per minute. On pelvic examination, her cervix is 3 cm dilated. Which of the following is the most likely diagnosis?

a. Premature rupture of membranes
b. Preterm labor
c. Cervical incompetence
d. Preterm contractions

Fischer, Conrad (2012-11-01). Master the Boards USMLE Step 2 CK (Page 452). Kaplan Medical Test Prep. Kindle Edition.

A

B. Preterm labor is diagnosed when there is a combination of contractions with cervical dilation. A premature rupture of membranes patient would have a history of a “gush of fluid” from the vagina. Patients with cervical incompetence do not have a history of contractions, but there is painless dilation of the cervix. Preterm contractions do not lead to cervical dilation.

Fischer, Conrad (2012-11-01). Master the Boards USMLE Step 2 CK (Page 452). Kaplan Medical Test Prep. Kindle Edition.

How well did you know this?
1
Not at all
2
3
4
5
Perfectly
37
Q

List the risk factors for preterm labor

A
  1. Premature rupture of membranes
  2. Multiple gestation
  3. Previous history of preterm labor
  4. Placental abruption
  5. Maternal factors
    • Uterine anatomical abnormalities
    • Infections (chorioamnionitis)
    • Preeclampsia
    • Intraadominal surgery

Fischer, Conrad (2012-11-01). Master the Boards USMLE Step 2 CK (Page 452). Kaplan Medical Test Prep. Kindle Edition.

How well did you know this?
1
Not at all
2
3
4
5
Perfectly
38
Q

List the circumstances under which preterm labor should not be stopped with tocolytics and delivery should occur

A
  • Maternal severe HTN (preeclampsia)
  • Maternal cardiac disease
  • Maternal cervical dilation of more than 4 cm
  • Maternal hemorrhage (abruptio placenta, DIC)
  • Fetal death
  • Chorioamnionitis
How well did you know this?
1
Not at all
2
3
4
5
Perfectly
39
Q

List the causes of late pregnancy bleeding (i.e., bleeding that occurs after 20 weeks’ gestation)

A
  • Abruptio placenta (painful)
  • Placenta previa (painless)
  • Vasa previa (painless)
  • Less commonly, lower genital tract lacerations and uterine rupture
  • bleeding: • Perform initial management: – Get the patient’s vitals

Fischer, Conrad (2012-09-22). Master the Boards: USMLE Step 3 (Kaplan Medical Usmle Master the Boards Step 3) (Kindle Locations 12464-12480). Kaplan Publishing. Kindle Edition.

How well did you know this?
1
Not at all
2
3
4
5
Perfectly
40
Q

List the initial steps in the management of late pregnancy bleeding

A
  1. Perform initial management:
    • Get the patient’s vitals
    • Place external fetal monitor
    • Start IV fluids with normal saline
  2. Order lab tests:
    • CBC
    • DIC workup (platelets, PT, PTT, fibrinogen, and D-dimer)
    • Type and cross-match
    • Obstetric ultrasound to rule out placenta previa
  3. Perform further steps in management:
    • Give blood transfusion for large volume loss
    • Place Foley catheter and measure urine output
    • Perform vaginal exam to rule out lacerations
    • Schedule delivery if fetus is in jeopardy or gestational age is ≥ 36 weeks

Fischer, Conrad (2012-09-22). Master the Boards: USMLE Step 3 (Kaplan Medical Usmle Master the Boards Step 3) (Kindle Locations 12477-12499). Kaplan Publishing. Kindle Edition.

How well did you know this?
1
Not at all
2
3
4
5
Perfectly
41
Q

True or False: Never perform a digital or speculum examination in a patient with late vaginal bleeding until a vaginal ultrasound first rules out placenta previa

Fischer, Conrad (2012-09-22). Master the Boards: USMLE Step 3 (Kaplan Medical Usmle Master the Boards Step 3) (Kindle Locations 12477-12499). Kaplan Publishing. Kindle Edition.

A

True

How well did you know this?
1
Not at all
2
3
4
5
Perfectly
42
Q

How do we know there is compromise in late pregnancy bleeding?

What condition most likely causes this and why?

A

Late decelerations and/or bradycardia

Vasa previa because the bleeding is from fetal circulation

How well did you know this?
1
Not at all
2
3
4
5
Perfectly
43
Q

List (1) diagnosis, (2) diagnostic test, (3) management and (4) complication based on the following presentation

  • Sudden onset vaginal bleeding
  • Pain
  • Hx of HTN
  • +/- Trauma
  • +/- Cocaine abuse

Fischer, Conrad (2012-09-22). Master the Boards: USMLE Step 3 (Kaplan Medical Usmle Master the Boards Step 3) (Kindle Locations 12502-12522). Kaplan Publishing. Kindle Edition.

A
  1. Abruptio placenta
  2. Abd/Pelvic US
  3. Mgnt:
    • Emergent c-section
    • Vaginal delivery may be attempted if ≥ 36 weeks and placenta is > 2 cm from internal os
    • Admit and observe if bleeding has stopped, vitals and fetal heart rate (FHR) stable, or < 34 weeks
  4. DIC. Amniotomy and induction of labor decrease the risk of DIC

Fischer, Conrad (2012-09-22). Master the Boards: USMLE Step 3 (Kaplan Medical Usmle Master the Boards Step 3) (Kindle Locations 12587-12590). Kaplan Publishing. Kindle Edition.

How well did you know this?
1
Not at all
2
3
4
5
Perfectly
44
Q

List (1) diagnosis, (2) diagnostic test, (3) management and (4) complication based on the following presentation

  • Sudden onset vaginal bleeding at rest or during activity without warning
  • No pain
  • +/- Trauma, coitis, or pelvic examination
  • +/- Multiparity
  • +/- Structural abnormalities (e.g. fibroids)
  • Advanced maternal age
A
  1. Placenta previa. Occurs when the placenta is implanted in the lower uterine segment. As the lower uterus stretches, placental villi dislodge and painless vaginal bleeding results
  2. Abd/pelvic US
  3. Mgnt:
    • Emergent c-section if patient/ fetus is deteriorating
    • Vaginal delivery if ≥ 36 weeks or continued bleeding
    • Admit and observe if bleeding has stopped, vitals and fetal heart rate (FHR) stable, or < 34 weeks
  4. Placenta accreta/increata/percreta → hysterectomy

Fischer, Conrad (2012-09-22). Master the Boards: USMLE Step 3 (Kaplan Medical Usmle Master the Boards Step 3) (Kindle Locations 12502-12522). Kaplan Publishing. Kindle Edition.

How well did you know this?
1
Not at all
2
3
4
5
Perfectly
45
Q

List (1) diagnosis, (2) diagnostic test, (3) management and (4) complication based on the following presentation

  • Rupture of membranes
  • Painless vaginal bleeding
  • Fetal bradycardia

Fischer, Conrad (2012-09-22). Master the Boards: USMLE Step 3 (Kaplan Medical Usmle Master the Boards Step 3) (Kindle Locations 12523-12544). Kaplan Publishing. Kindle Edition.

A
  1. Vasa Previa
  2. Abd/pelvic US
  3. Emergency CS
  4. Fetal exsanguination

Fischer, Conrad (2012-09-22). Master the Boards: USMLE Step 3 (Kaplan Medical Usmle Master the Boards Step 3) (Kindle Locations 12523-12544). Kaplan Publishing. Kindle Edition.

How well did you know this?
1
Not at all
2
3
4
5
Perfectly
46
Q

List (1) diagnosis, (2) diagnostic test, (3) management and (4) complication based on the following presentation

  • Hx of uterine scar
  • Sudden onset abd pain
  • Sudden vaginal bleeding
  • Loss of electronic fetal heart rate
  • Loss of electronic uterine contractions
  • Recession of the fetal head
  • +/- Myomectomy for fibroids
  • +/- Excessive oxytocin
  • +/- Grand multiparity

Fischer, Conrad (2012-09-22). Master the Boards: USMLE Step 3 (Kaplan Medical Usmle Master the Boards Step 3) (Kindle Locations 12523-12544). Kaplan Publishing. Kindle Edition.

A
  1. Uterine rupture
  2. N/A
  3. Immediate surgery and delivery
  4. Hysterectomy for uncontrolled bleeding
How well did you know this?
1
Not at all
2
3
4
5
Perfectly
47
Q

A 28-year-old woman presents at 36 weeks’ gestation with rupture of membranes. On examination she is found to have 7 cm cervical dilatation. She received all of her prenatal care, and her only complication was a course of antibiotics for asymptomatic bacteriuria. GBS screening was negative. Her first baby was hospitalized for 10 days after delivery for GBS pneumonia and sepsis. What is the most appropriate management?

a. Administer intrapartum IV penicillin
b. Administer intramuscular azithromycin
c. Rescreen for group B streptococci
d. Schedule cesarean section
e. No intervention is needed

Fischer, Conrad (2012-09-22). Master the Boards: USMLE Step 3 (Kaplan Medical Usmle Master the Boards Step 3) (Kindle Locations 12602-12613). Kaplan Publishing. Kindle Edition.

A

A. Intrapartum IV penicillin is indicated because the patient’s previous birth was complicated with neonatal GBS sepsis.

Fischer, Conrad (2012-09-22). Master the Boards: USMLE Step 3 (Kaplan Medical Usmle Master the Boards Step 3) (Kindle Locations 12602-12613). Kaplan Publishing. Kindle Edition.

How well did you know this?
1
Not at all
2
3
4
5
Perfectly
48
Q

True or False:

  1. Thirty percent of women have asymptomatic vaginal colonization with GBS
  2. Vertical transmission results in pneumonia and sepsis in the neonate within hours to days of birth
  3. There is a 50 percent mortality rate with neonatal infection
  4. GBS-related meningitis occurs after the first week and is a hospital-acquired infection that is related to vertical transmission

Fischer, Conrad (2012-09-22). Master the Boards: USMLE Step 3 (Kaplan Medical Usmle Master the Boards Step 3) (Kindle Locations 12613-12632). Kaplan Publishing. Kindle Edition.

A
  1. True
  2. True
  3. True
  4. False. The infection is unrelated to vertical transmission
How well did you know this?
1
Not at all
2
3
4
5
Perfectly
49
Q

List the Rx of GBS

A
  • Intrapartum IV penicillin G
  • Penicillin allergy: IV cefazolin, clindamycin, or erythromycin

Fischer, Conrad (2012-09-22). Master the Boards: USMLE Step 3 (Kaplan Medical Usmle Master the Boards Step 3) (Kindle Locations 12613-12632). Kaplan Publishing. Kindle Edition.

How well did you know this?
1
Not at all
2
3
4
5
Perfectly
50
Q

Indications for Rx of GBS

Fischer, Conrad (2012-09-22). Master the Boards: USMLE Step 3 (Kaplan Medical Usmle Master the Boards Step 3) (Kindle Locations 12613-12632). Kaplan Publishing. Kindle Edition.

A
  1. GBS (+) urine, cervical, or vaginal culture at any time during pregnancy
  2. Presence of high-risk factors:
    • Preterm delivery
    • Membrane rupture > 18 hours
    • Maternal fever
    • Previous baby with GBS sepsis

Fischer, Conrad (2012-09-22). Master the Boards: USMLE Step 3 (Kaplan Medical Usmle Master the Boards Step 3) (Kindle Locations 12613-12632). Kaplan Publishing. Kindle Edition.

How well did you know this?
1
Not at all
2
3
4
5
Perfectly
51
Q

Exceptions to antibiotic use in GBS

Fischer, Conrad (2012-09-22). Master the Boards: USMLE Step 3 (Kaplan Medical Usmle Master the Boards Step 3) (Kindle Locations 12634-12654). Kaplan Publishing. Kindle Edition.

A
  • Planned c-section without rupture of membranes (even if culture is [+])
  • Culture (+) on a previous pregnancy, but culture (–) in the current pregnancy

Fischer, Conrad (2012-09-22). Master the Boards: USMLE Step 3 (Kaplan Medical Usmle Master the Boards Step 3) (Kindle Locations 12634-12654). Kaplan Publishing. Kindle Edition.

How well did you know this?
1
Not at all
2
3
4
5
Perfectly
52
Q

Interpret the following results:

  1. IgG antibodies to GBS in the mother
  2. IgM antibodies to GBS in the mother

Fischer, Conrad (2012-09-22). Master the Boards: USMLE Step 3 (Kaplan Medical Usmle Master the Boards Step 3) (Kindle Locations 12613-12632). Kaplan Publishing. Kindle Edition.

A
  1. Past exposure and are protective
  2. Suggests recent exposure and risk of exposure to the fetus.

Fischer, Conrad (2012-09-22). Master the Boards: USMLE Step 3 (Kaplan Medical Usmle Master the Boards Step 3) (Kindle Locations 12613-12632). Kaplan Publishing. Kindle Edition.

How well did you know this?
1
Not at all
2
3
4
5
Perfectly
53
Q

Toxoplasmosis in the infant:

  1. Causative organism
  2. Risk factors (RF)
  3. Prevention
  4. Rx
A
  1. Toxoplasma gondii
  2. RF:
    • Patient handling cat feces or litter boxes
    • Drinking raw goat milk
    • Eating raw meat.
  3. Prevention: Avoid risk factors
  4. Pyrimethamine and sulfadiazine

Fischer, Conrad (2012-09-22). Master the Boards: USMLE Step 3 (Kaplan Medical Usmle Master the Boards Step 3) (Kindle Locations 12634-12654). Kaplan Publishing. Kindle Edition.

How well did you know this?
1
Not at all
2
3
4
5
Perfectly
54
Q

Diagnosis:

  1. Chorioretinitis
  2. Intracranial calcifications
  3. Hydrocephalus
A

Congenital toxoplasmosis

Fischer, Conrad (2012-09-22). Master the Boards: USMLE Step 3 (Kaplan Medical Usmle Master the Boards Step 3) (Kindle Locations 12634-12654). Kaplan Publishing. Kindle Edition.

How well did you know this?
1
Not at all
2
3
4
5
Perfectly
55
Q

A 29-year-old woman (G2 P1) is at 34 weeks’ gestation. She complains of uterine contractions every 5 minutes. Her previous records are not available. During the last few days, she has developed diffuse pruritic vesicles on her neck, which appear to be also developing on her chest and breasts. She has a fever and complains of malaise. Which of the following is the next step in management?

a. Oral acyclovir
b. RhoGAM
c. Varicella zoster antibody assay
d. Varicella vaccine
e. Varicella zoster immunoglobulin

Fischer, Conrad (2012-09-22). Master the Boards: USMLE Step 3 (Kaplan Medical Usmle Master the Boards Step 3) (Kindle Locations 12661-12673). Kaplan Publishing. Kindle Edition.

A

C. Varicella antibodies are seen in 90 percent of pregnant women due to prior infection. Varicella antibodies are protective, and no therapy is necessary

Fischer, Conrad (2012-09-22). Master the Boards: USMLE Step 3 (Kaplan Medical Usmle Master the Boards Step 3) (Kindle Locations 12661-12673). Kaplan Publishing. Kindle Edition.

How well did you know this?
1
Not at all
2
3
4
5
Perfectly
56
Q
  1. Diagnosis:
  • Zigzag” skin lesions
  • Limb hypoplasia
  • Microcephaly
  • Microphthalmia
  • Chorioretinitis
  • Cataracts.
  1. Prevention of above
  2. Rx of above
A
  1. Neonatal varicella infection. Transplacental infection results from primary varicella infection in the mother (25– 40 percent infection rate). The greatest risk to the fetus is if a rash appears in the mother between 5 days antepartum and 2 days postpartum.
  2. Prevention
    • Vaccination: Live-attenuated varicella virus (Varivax III) to nonpregnant women
    • Postexposure prophylaxis: VariZIG (antivaricella antibodies) or varicella zoster immunoglobulin within 96 hours of exposure. VariZIG/ VZIG does not prevent infection but only attenuates the clinical effects of the virus.
  3. Treatment
    • Maternal varicella: VariZIG to mother and neonate
    • Congenital varicella: VariZIG and IV acyclovir to the neonate

Fischer, Conrad (2012-09-22). Master the Boards: USMLE Step 3 (Kaplan Medical Usmle Master the Boards Step 3) (Kindle Locations 12674-12691). Kaplan Publishing. Kindle Edition.

How well did you know this?
1
Not at all
2
3
4
5
Perfectly
57
Q

Diagnosis:

  • Congenital deafness (most common sequelae)
  • Ccongenital heart disease (e.g., patent ductus arteriosus, or PDA)
  • Cataracts
  • Mental retardation
  • Hhepatosplenomegaly
  • Tthrombocytopenia
  • “Blueberry muffin” rash

Prevention?

Fischer, Conrad (2012-09-22). Master the Boards: USMLE Step 3 (Kaplan Medical Usmle Master the Boards Step 3) (Kindle Locations 12674-12691). Kaplan Publishing. Kindle Edition.

A

Diagnosis: Congenital rubella

Prevetion:

  • Perform first-trimester screening and have mother avoid infected individuals
  • Immunize seronegative women after delivery
  • No postexposure prophylaxis is available

Fischer, Conrad (2012-09-22). Master the Boards: USMLE Step 3 (Kaplan Medical Usmle Master the Boards Step 3) (Kindle Locations 12692-12713). Kaplan Publishing. Kindle Edition.

How well did you know this?
1
Not at all
2
3
4
5
Perfectly
58
Q

A 24-year-old child-care worker is 29 weeks pregnant and is currently working. One of the children was diagnosed with rubella last week. Rubella antigen testing is performed and her IgG titer is negative. What is the risk of neonatal transmission in this patient? What is the next step in management?

a. Give anti-rubella antibodies
b. Give betamethasone
c. Give rubella vaccine now
d. Give rubella vaccine after delivery
e. Ultrasound of the fetus

Fischer, Conrad (2012-09-22). Master the Boards: USMLE Step 3 (Kaplan Medical Usmle Master the Boards Step 3) (Kindle Locations 12692-12713). Kaplan Publishing. Kindle Edition.

A

D. There is no postexposure prophylaxis available, and immunization during pregnancy is contraindicated (live vaccine). The only correct management is to await normal delivery and give vaccination to the mother after delivery

Fischer, Conrad (2012-09-22). Master the Boards: USMLE Step 3 (Kaplan Medical Usmle Master the Boards Step 3) (Kindle Locations 12692-12713). Kaplan Publishing. Kindle Edition.

How well did you know this?
1
Not at all
2
3
4
5
Perfectly
59
Q

Diagnosis:

  • Most common congenital viral syndrome in the United States
  • Most common cause of sensorineural deafness in children
  • Spread by infected body fluid secretions
  • The greatest risk for vertical transmission occurs with primary infection (infection rate is 50 percent)
  • Most mothers develop asymptomatic infections or describe mild, mononucleosis-like symptoms.

Fischer, Conrad (2012-09-22). Master the Boards: USMLE Step 3 (Kaplan Medical Usmle Master the Boards Step 3) (Kindle Locations 12692-12713). Kaplan Publishing. Kindle Edition.

A

Cytomegalovirus (CMV)/Congenital CMV

About 10% of infants with congenital CMV infection are symptomatic at birth

Fischer, Conrad (2012-09-22). Master the Boards: USMLE Step 3 (Kaplan Medical Usmle Master the Boards Step 3) (Kindle Locations 12692-12713). Kaplan Publishing. Kindle Edition.

How well did you know this?
1
Not at all
2
3
4
5
Perfectly
60
Q

Diagnosis:

  • Intrauterine growth restriction (IUGR)
  • Prematurity
  • Microcephaly
  • Jjaundice
  • Petechiae
  • Hepatosplenomegaly
  • Periventricular calcifications
  • Chorioretinitis
  • Pneumonitis
A

Congenital CMV

Fischer, Conrad (2012-09-22). Master the Boards: USMLE Step 3 (Kaplan Medical Usmle Master the Boards Step 3) (Kindle Locations 12713-12729). Kaplan Publishing. Kindle Edition.

How well did you know this?
1
Not at all
2
3
4
5
Perfectly
61
Q

Interpretation of results:

  1. Maternal IgG (+) / IgM (–)
  2. Maternal IgG (+) / IgM (+)
  3. Maternal IgG (–) / IgM (+) indicates recent infection.
A
  1. Indicates past exposure and no risk for primary infection
  2. Indicates recent infection
  3. Indicates recent infection

Perform a viral culture from urine or other body fluids in first 2 weeks of life and CMV DNA-PCR

Fischer, Conrad (2012-09-22). Master the Boards: USMLE Step 3 (Kaplan Medical Usmle Master the Boards Step 3) (Kindle Locations 12713-12729). Kaplan Publishing. Kindle Edition.

How well did you know this?
1
Not at all
2
3
4
5
Perfectly
62
Q
  1. Prevention of CMV
  2. Rx of CMV
A
  1. Prevention:
    • Follow universal precautions with all body fluids.
    • Avoid transfusion with CMV-positive blood
  2. Treatment:
    • Antiviral therapy with ganciclovir. This prevents viral shedding and prevents hearing loss but does not cure the infection

Fischer, Conrad (2012-09-22). Master the Boards: USMLE Step 3 (Kaplan Medical Usmle Master the Boards Step 3) (Kindle Locations 12713-12729). Kaplan Publishing. Kindle Edition.

How well did you know this?
1
Not at all
2
3
4
5
Perfectly
63
Q

A 21-year-old multipara is admitted to the birthing unit at 39 weeks gestation in active labor at 6 cm dilation. Membranes are intact. She has a history of genital herpes preceding the pregnancy. Her last outbreak was 8 weeks ago. She now complains of pain and pruritus. On examination, she had localized, painful, ulcerative lesions on her right vaginal wall. Which of the following is the next step in management?

a. Administer IV acyclovir
b. Administer terbutaline
c. Obtain culture of ulcer
d. Proceed with vaginal delivery
e. Schedule cesarean section

Fischer, Conrad (2012-09-22). Master the Boards: USMLE Step 3 (Kaplan Medical Usmle Master the Boards Step 3) (Kindle Locations 12729-12748). Kaplan Publishing. Kindle Edition.

A

E. Active genital herpes is an indication for cesarean section. The only other infection-related indication for cesarean section is HIV sero-positive status of the mother

Fischer, Conrad (2012-09-22). Master the Boards: USMLE Step 3 (Kaplan Medical Usmle Master the Boards Step 3) (Kindle Locations 12729-12748). Kaplan Publishing. Kindle Edition.

How well did you know this?
1
Not at all
2
3
4
5
Perfectly
64
Q

True or False:

  1. The most common cause of transmission is contact with maternal genital lesions during an active HSV episode.
  2. Transplacental infection can also occur with primary infections during pregnancy (50 percent risk).
  3. Greatest risk is primary infection in the third trimester.
  4. Neonatal infection acquired during delivery has 50 percent mortality rate.
  5. Surviving infants develop meningoencephalitis, mental retardation, pneumonia, hepatosplenomegaly, jaundice, and petechiae.

Fischer, Conrad (2012-09-22). Master the Boards: USMLE Step 3 (Kaplan Medical Usmle Master the Boards Step 3) (Kindle Locations 12729-12748). Kaplan Publishing. Kindle Edition.

A
  1. True
  2. True
  3. True
  4. True
  5. True

Fischer, Conrad (2012-09-22). Master the Boards: USMLE Step 3 (Kaplan Medical Usmle Master the Boards Step 3) (Kindle Locations 12729-12748). Kaplan Publishing. Kindle Edition.

How well did you know this?
1
Not at all
2
3
4
5
Perfectly
65
Q

Diagnostic test for HSV

Rx of HSV

A

Diagnostic Testing (+) HSV culture from vesicle fluid or ulcer or HSV PCR of maternal blood (not screened routinely)

Rx: Acyclovir

Fischer, Conrad (2012-09-22). Master the Boards: USMLE Step 3 (Kaplan Medical Usmle Master the Boards Step 3) (Kindle Locations 12729-12748). Kaplan Publishing. Kindle Edition.

How well did you know this?
1
Not at all
2
3
4
5
Perfectly
66
Q

Prevention of HSV

A
  • Perform c-section in women with lesions suspicious for active genital HSV at the time of labor
  • Do not use fetal scalp electrodes for monitoring (increased risk of HSV transmission)
  • Manage expectantly for premature rupture of membranes (PROM) or prolonged rupture of membranes (ROM) (i.e., > 8–12 hours)
  • Advise standard precautions (avoid intercourse if partner has active lesions, avoid oral sex in presence of oral lesions, avoid kissing neonate in presence of oral lesions)

Fischer, Conrad (2012-09-22). Master the Boards: USMLE Step 3 (Kaplan Medical Usmle Master the Boards Step 3) (Kindle Locations 12749-12766). Kaplan Publishing. Kindle Edition.

67
Q

A 24-year-old HIV positive female (G2 P1) presents in her 16th week of pregnancy. Her previous child was diagnosed HIV positive after vaginal delivery. What is the most effective method of decreasing the risk of vertical transmission?

a. Avoidance of artificial rupture of membranes
b. Avoidance of breastfeeding
c. Antiretroviral triple therapy
d. Cesarean section
e. Zidovudine monotherapy

Fischer, Conrad (2012-09-22). Master the Boards: USMLE Step 3 (Kaplan Medical Usmle Master the Boards Step 3) (Kindle Locations 12749-12766). Kaplan Publishing. Kindle Edition.

A

C. All of the strategies are recommended, however. Zidovudine (ZDV) monotherapy is not as effective as triple therapy in decreasing the risk of HIV transmission to the fetus (25 percent to 8 percent). Triple antiretroviral therapy is indicated for more effective management of HIV in the mother to drive the viral load to < 1,000. ZDV monotherapy alone is never indicated. Cesarean section (before rupture of membranes), avoidance of breastfeeding or intrapartum invasive procedures (artificial ROM, fetal scalp electrodes) also decreases transmission rate. Combination of all of the above strategies listed above reduces the transmission rate to 1 percent.

Fischer, Conrad (2012-09-22). Master the Boards: USMLE Step 3 (Kaplan Medical Usmle Master the Boards Step 3) (Kindle Locations 12749-12766). Kaplan Publishing. Kindle Edition.

68
Q

HIV

  1. True or False: Major route of vertical transmission is contact with infected genital secretions at the time of vaginal delivery
  2. What is the vertical transmission rate without Rx
  3. True or False: . Elective cesarean is of most benefit in women with low CD4 counts and high RNA viral loads (> 1,000)
  4. True or False: All neonates of HIV-positive women will have positive HIV tests from transplacental passive IgG passage
  5. True or False: Zidovudine monotherapy is no longer indicated for anyone
  6. Continue antiretrovirals in pregnancy for HIV (+) patients with low CD4 counts and/ or high viral load.
A
  1. True
  2. 25– 30 percent
  3. True
  4. True
  5. True
  6. True

Fischer, Conrad (2012-09-22). Master the Boards: USMLE Step 3 (Kaplan Medical Usmle Master the Boards Step 3) (Kindle Locations 12766-12786). Kaplan Publishing. Kindle Edition.

69
Q

Outline Prevention and Rx of HIV in pregnancy

A
  1. Triple-drug therapy (which must include ZDV):
    • Starting at 14 weeks for mothers who have high CD4 counts and do not need medication for their own health and continuing throughout pregnancy
    • Intravenous intrapartum ZDV
    • Combination ZDV-based ART for 6 weeks after delivery
  2. Give the infant prophylaxis against pneumocystis pneumonia (trimethoprim-sulfamethoxazole) and continue for 6 weeks after AZT therapy has been completed
  3. Schedule C-section at 38 weeks unless < 1,000 viral copies/mL
  4. Advise the mother not to breastfeed
  5. Avoid invasive procedures (e.g., artificial rupture of membranes, fetal scalp electrodes).

HIV-infected pregnant women should receive ART therapy regardless of HIV RNA level

Do a c-section of the mother’s viral load is > 1,000 at the time of delivery.

Fischer, Conrad (2012-09-22). Master the Boards: USMLE Step 3 (Kaplan Medical Usmle Master the Boards Step 3) (Kindle Locations 12766-12786). Kaplan Publishing. Kindle Edition.

70
Q
  1. True or False: Prior syphilis infection confers immunity
  2. True or False: Transplacental infection results from primary and secondary infection (60 percent risk of transmission)
  3. True or False: The lowest risk of transmission is with latent or tertiary infection.
A
  1. False
  2. True
  3. True

Fischer, Conrad (2012-09-22). Master the Boards: USMLE Step 3 (Kaplan Medical Usmle Master the Boards Step 3) (Kindle Locations 12789-12804). Kaplan Publishing. Kindle Edition.

71
Q

List the symptoms/outcomes of early acquired (first trimester) congenital syphilis

A
  • Nonimmune hydrops fetalis
  • Maculopapular or vesicular peripheral rash
  • Anemia, thrombocytopenia, and hepatosplenomegaly
  • Large and edematous placenta
  • Perinatal mortality rates ~ 50 percent

Fischer, Conrad (2012-09-22). Master the Boards: USMLE Step 3 (Kaplan Medical Usmle Master the Boards Step 3) (Kindle Locations 12789-12804). Kaplan Publishing. Kindle Edition.

72
Q

List the symptoms/outcomes of late acquired congenital syphilis

A

Hutchinson teeth

“Mulberry” molars

“Saddle” nose

“Saber” shins

Deafness (cranial nerve 8 palsy)

Fischer, Conrad (2012-09-22). Master the Boards: USMLE Step 3 (Kaplan Medical Usmle Master the Boards Step 3) (Kindle Locations 12789-12804). Kaplan Publishing. Kindle Edition.

73
Q
  1. True or False: C-section will not prevent vertical transmission of syphilis, because it happens through the placenta before birth
  2. True or False: Always order an HIV test in any pregnant patient who has tested positive for an STD

Fischer, Conrad (2012-09-22). Master the Boards: USMLE Step 3 (Kaplan Medical Usmle Master the Boards Step 3) (Kindle Locations 12804-12820). Kaplan Publishing. Kindle Edition.

A
  1. True
  2. True
74
Q

Diagnosis and Rx of syphilis in pregnancy

A

Diagnostic Testing

  • VDRT or RPR screening in first trimester. Confirm (+) screen with FTA-ABS or MHA-TP
  • Screening test will be falsely negative in primary syphilis. – When the case describes a woman with a painless genital ulcer, order darkfield microscopy for diagnosis of primary syphilis

Treatment

  • Benzathine penicillin IM × 1 for (+) mothers
  • Penicillin allergy: Oral desensitization followed by full dose benzathine penicillin

Fischer, Conrad (2012-09-22). Master the Boards: USMLE Step 3 (Kaplan Medical Usmle Master the Boards Step 3) (Kindle Locations 12804-12820). Kaplan Publishing. Kindle Edition.

75
Q

A 34-year-old multigravida presents for prenatal care in the second trimester. She admits to a past history of substance abuse but states she has been clean for 6 months. With her second pregnancy, she experienced a preterm delivery at 34 weeks’ gestation of a male neonate who died within the first day of life. She states that at delivery, the baby was swollen with skin lesions and that the placenta was very large. She was treated with antibiotics, but she does not remember their name or other details. On a routine prenatal panel with this current pregnancy, she is found to have a positive VDRL test. What is the next step in management?

a. FTA-ABS
b. Intramuscular penicillin
c. Lupus anticoagulant
d. Oral penicillin
e. RPR
f. Ultrasound

Fischer, Conrad (2012-09-22). Master the Boards: USMLE Step 3 (Kaplan Medical Usmle Master the Boards Step 3) (Kindle Locations 12804-12820). Kaplan Publishing. Kindle Edition.

A

A. The next step after any positive screening test is the confirmatory test before starting therapy. FTA-ABS or MHA-TP are the confirmatory tests for syphilis. Once syphilis is confirmed, the most appropriate management is intramuscular penicillin

Fischer, Conrad (2012-09-22). Master the Boards: USMLE Step 3 (Kaplan Medical Usmle Master the Boards Step 3) (Kindle Locations 12804-12820). Kaplan Publishing. Kindle Edition

76
Q
  1. Ways of acquisition of neonatal hepatitis B virus (HBV) infection
  2. How many percent of neonates who get infected develop chronic hepatitis
A
  1. Neonatal HBV:
    • In the third trimester
    • Ingestion of infected genital secretions during vaginal delivery
  2. 80 percent compared with only 10 percent of infected adults.

Fischer, Conrad (2012-09-22). Master the Boards: USMLE Step 3 (Kaplan Medical Usmle Master the Boards Step 3) (Kindle Locations 12821-12846). Kaplan Publishing. Kindle Edition.

77
Q

A 29-year-old multigravida was found on routine prenatal laboratory testing to be positive for hepatitis B surface antigen. She is an intensive care unit nurse. She received 2 units of packed red blood cells 2 years ago after experiencing postpartum hemorrhage with her last pregnancy. Which of the following indicates the greatest risk of transmission?

a. Anti-HBc
b. Anti-HBs
c. HBe Ag
d. HBs Ag
e. IgM anti-HBc

Fischer, Conrad (2012-09-22). Master the Boards: USMLE Step 3 (Kaplan Medical Usmle Master the Boards Step 3) (Kindle Locations 12821-12846). Kaplan Publishing. Kindle Edition.

A

C. Mothers who are (+) for HBsAg, anti-HBc, and IgM anti-HBc are acutely infected. There is only a 10 percent vertical transmission risk. Mothers who are also (+) for HBeAg have an 80 percent risk of transmission to fetus. Anti-HBs (antibody to surface antigen) indicates immunity to infection from previous immunization.

Fischer, Conrad (2012-09-22). Master the Boards: USMLE Step 3 (Kaplan Medical Usmle Master the Boards Step 3) (Kindle Locations 12821-12846). Kaplan Publishing. Kindle Edition.

78
Q

Prevention of HBV infection in the neonate

A
  1. Hepatitis B infection is not an indication for cesarean delivery.
  2. Avoid invasive procedures during pregnancy (e.g., amniocentesis)
  3. Breastfeeding is not contraindicated after the neonate has received active immunization and HBIG
  4. Immunization:
    • HBsAg-negative: Give active immunization during pregnancy
    • Postexposure prophylaxis for the mother: HBIG (antibodies to hepatitis B) passive immunization
  5. Treatment:
    • Hepatitis immunization and HBIG in neonate
    • Chronic HBV can be treated with either interferon or lamivudine

Fischer, Conrad (2012-09-22). Master the Boards: USMLE Step 3 (Kaplan Medical Usmle Master the Boards Step 3) (Kindle Locations 12846-12866). Kaplan Publishing. Kindle Edition.

79
Q

List the contraindications to breastfeeding

A
  1. Infections in the mother:
    • HIV
    • Active tuberculosis
    • HTLV-1
    • Herpes simplex if there is a lesion on the breast
  2. Use of drugs/ medications
  3. Drugs of abuse (except cigarettes, alcohol)
  4. Cytotoxic medications (e.g., methotrexate, cyclosporine)
  5. Condition of the infant – Galactosemia

Fischer, Conrad (2012-09-22). Master the Boards: USMLE Step 3 (Kaplan Medical Usmle Master the Boards Step 3) (Kindle Locations 12850-12860). Kaplan Publishing. Kindle Edition.

80
Q

Hypertension (BP ≥ 140/ 90 mm Hg) during pregnancy can be classified as chronic hypertension or gestational hypertension. Both types of hypertension predispose the mother and the fetus to more serious conditions

List the differential diagnosis of hypertension that is accompanied by signs and symptoms of end-organ damage or neurological sequelae

Fischer, Conrad (2012-09-22). Master the Boards: USMLE Step 3 (Kaplan Medical Usmle Master the Boards Step 3) (Kindle Locations 12867-12887). Kaplan Publishing. Kindle Edition.

A
  • Preeclampsia
  • Eclampsia
  • HELLP syndrome

Fischer, Conrad (2012-09-22). Master the Boards: USMLE Step 3 (Kaplan Medical Usmle Master the Boards Step 3) (Kindle Locations 12867-12887). Kaplan Publishing. Kindle Edition.

81
Q

Warning signs of maternal jeopardy in HTN in pregnancy

A
  1. Hallmark symptoms:
    • Headache
    • Epigastric pain
    • Changes in vision
  2. Signs:
    • Pulmonary edema
    • Oliguria (Peripheral edema is not a warning sign.)
  3. Labs:
    • Thrombocytopenia
    • Elevated liver enzymes

Fischer, Conrad (2012-09-22). Master the Boards: USMLE Step 3 (Kaplan Medical Usmle Master the Boards Step 3) (Kindle Locations 12867-12887). Kaplan Publishing. Kindle Edition.

82
Q

Risk of sustained maternal HTN on the fetus

A

IUGR

Hypoxia

Abruptio placenta

Fischer, Conrad (2012-09-22). Master the Boards: USMLE Step 3 (Kaplan Medical Usmle Master the Boards Step 3) (Kindle Locations 12888-12906). Kaplan Publishing. Kindle Edition.

83
Q

Define:

Chronic HTN

Gestational HTN

Preeclampsia

Mild preeclampsia

Severe preeclampsia

A

Chronic hypertension is the diagnosis when there is a history of elevated blood pressure before pregnancy or before 20 weeks’ gestation

Gestational hypertension is the diagnosis when blood pressure develops after 20 weeks’ gestation and returns to normal baseline by 6 weeks post-partum. It occurs more commonly in multifetal pregnancy

Preeclampsia is the diagnosis when there is proteinuria and/ or presence of “warning signs.”

Mild preeclampsia is indicated with:

  • Sustained BP elevation > 140/ 90 mm Hg
  • Proteinuria of 1– 2 + (on dipstick) or > 300 mg (on a 24-hour urine)

Severe preeclampsia is indicated by mild preeclampsia plus one of the following:

  • Sustained BP elevation > 160/110 mm Hg
  • Proteinuria of 3– 4 + (on dipstick) or > 5 g (on 24-hour urine)
  • Presence of “warning signs”

Fischer, Conrad (2012-09-22). Master the Boards: USMLE Step 3 (Kaplan Medical Usmle Master the Boards Step 3) (Kindle Locations 12888-12906). Kaplan Publishing. Kindle Edition.

84
Q

List the risk factors for preeclampsia

A
  • Primiparas are most at risk.
  • Multiple gestation
  • Hydatidi-form mole
  • Diabetes mellitus
  • Age extremes
  • Chronic hypertension
  • Chronic renal disease

Fischer, Conrad (2012-09-22). Master the Boards: USMLE Step 3 (Kaplan Medical Usmle Master the Boards Step 3) (Kindle Locations 12906-12927). Kaplan Publishing. Kindle Edition.

85
Q

A 19-year-old primigravida presents at 32 weeks’ gestation for routine follow-up. She denies headache, epigastric pain, or visual disturbances. She has gained 2 pounds since her last visit 2 weeks ago. On examination, her blood pressure is 155/ 95, which is persistent on repeat BP check 10 minutes later. She has only trace pedal edema. Which of the following is the next step in management?

a. Begin methyldopa
b. Begin labetalol
c. Perform an electrocardiogram
d. Perform a fetal ultrasound
e. Perform urinalysis

Fischer, Conrad (2012-09-22). Master the Boards: USMLE Step 3 (Kaplan Medical Usmle Master the Boards Step 3) (Kindle Locations 12906-12927). Kaplan Publishing. Kindle Edition.

A

E. Always rule out preeclampsia in a hypertensive pregnant patient. Even if she is asymptomatic, proteinuria indicates preeclampsia and a worse prognosis

Seizure disorder is not a risk factor for eclampsia.

Fischer, Conrad (2012-09-22). Master the Boards: USMLE Step 3 (Kaplan Medical Usmle Master the Boards Step 3) (Kindle Locations 12906-12927). Kaplan Publishing. Kindle Edition.

86
Q

Define:

Chronic hypertension with superimposed preeclampsia

Eclampsia

HELLP

A

Chronic hypertension with superimposed preeclampsia is the diagnosis when there is chronic hypertension with increasingly severe hypertension, proteinuria, and/ or “warning signs.”

Eclampsia is the diagnosis when the case describes unexplained grand mal seizures in a hypertensive and/ or proteinuric pregnant woman in the last half of pregnancy. Patients present with same signs and symptoms as in pre-eclampsia with the addition of unexplained tonic-clonic seizures. Seizures from severe diffuse cerebral vasospasm cause cerebral perfusion deficits and edema.

HELLP syndrome is the diagnosis when there is hemolysis (H), elevated liver (EL) enzymes, and low platelets (LP).

Fischer, Conrad (2012-09-22). Master the Boards: USMLE Step 3 (Kaplan Medical Usmle Master the Boards Step 3) (Kindle Locations 12906-12927). Kaplan Publishing. Kindle Edition.

87
Q

Outline the diagnostic tests to be done in HTN in pregnancy

A
  1. CBC
    • ↑ hemoglobin, ↑ hematocrit
  2. Chem-12 panel
    • ↑ blood urea nitrogen (BUN), ↑ serum creatinine, and ↑ serum uric acid
  3. Coagulation panel
    • DIC, elevated liver enzymes (severe preeclampsia)
  4. Urinalysis with urinary protein
    • Proteinuria

Fischer, Conrad (2012-09-22). Master the Boards: USMLE Step 3 (Kaplan Medical Usmle Master the Boards Step 3) (Kindle Locations 12927-12946). Kaplan Publishing. Kindle Edition.

88
Q

Rx of HTN in pregnancy

Blood pressure control

A

The only definitive cure is delivery and removal of all fetal-placental tissue

Blood pressure control:

  1. Don’t treat unless BP > 160/ 100 mm Hg (antihypertensives decrease uteroplacental blood flow)
  2. Goal SBP is 140– 150 mm Hg and DBP is 90– 100 mm Hg
  3. Maintenance therapy:
    • First line therapy is methyldopa
    • Second line therapy is β-blockers (labetalol, atenolol). Watch out for intrauterine growth restriction (IUGR), which is associated with β-blocker use in pregnancy
  4. Acutely elevated BP/ treatment of severe preeclampsia or eclampsia:
    • Intravenous hydralazine or labetalol
  5. Never give ACE inhibitors or start thiazide diuretics during pregnancy

Fischer, Conrad (2012-09-22). Master the Boards: USMLE Step 3 (Kaplan Medical Usmle Master the Boards Step 3) (Kindle Locations 12947-12974). Kaplan Publishing. Kindle Edition.

89
Q

HTN in pregnancy

Seizure management and prophylaxis

Monitoring

A

Seizure management and prophylaxis:

  • Protect the patient’s airway and tongue
  • Give IV MgSO4 (magnesium sulfate) bolus for seizure and infusion for continued prophylaxis

Monitoring:

  • Serial sonograms (evaluate for intrauterine growth restriction [IUGR])
  • Serial BP monitoring and urine protein

Fischer, Conrad (2012-09-22). Master the Boards: USMLE Step 3 (Kaplan Medical Usmle Master the Boards Step 3) (Kindle Locations 12947-12974). Kaplan Publishing. Kindle Edition.

90
Q

HTN in pregnancy

Mgnt of labor

A
  • Induce labor if ≥ 36 weeks with mild preeclampsia: attempt vaginal delivery with IV oxytocin if mother and fetus are stable
  • Aggressive, prompt delivery is the best step for severe/ superimposed pre-eclampsia or eclampsia at any gestational age
  • Give intrapartum IV MgSO4 and hydralazine and/ or labetalol to manage BP

Fischer, Conrad (2012-09-22). Master the Boards: USMLE Step 3 (Kaplan Medical Usmle Master the Boards Step 3) (Kindle Locations 12947-12974). Kaplan Publishing. Kindle Edition.

91
Q

A 32-year-old multigravida at 36 weeks’ gestation was found to have BP 160/ 105 on routine prenatal visit. Previous BP readings were normal. She complained of some right-upper-quadrant abdominal pain. Urinalysis showed 3 + proteinuria. She is emergently induced for labor and delivers an 8 lb. 3 oz. boy. Two days after delivery, routine labs reveal elevated total bilirubin, lactate dehydrogenase, alanine aminotransferase (ALT), and aspartate aminotransferase (AST). Platelet count is 85,000. Postpartum evaluation reveals that she has no complaints of headache or visual changes. Which of the following is the most likely diagnosis?

a. Cholecystitis
b. HELLP syndrome
c. Hepatitis
d. Gestational thrombocytopenia
e. Preeclampsia

Fischer, Conrad (2012-09-22). Master the Boards: USMLE Step 3 (Kaplan Medical Usmle Master the Boards Step 3) (Kindle Locations 12974-12995). Kaplan Publishing. Kindle Edition.

A

B. Patient has evidence of hemolysis (elevated LDH), elevated liver enzymes, and thrombocytopenia

Fischer, Conrad (2012-09-22). Master the Boards: USMLE Step 3 (Kaplan Medical Usmle Master the Boards Step 3) (Kindle Locations 12974-12995). Kaplan Publishing. Kindle Edition.

92
Q

List the features of gestational thrombocytopenia

A
  • Most common cause of thrombocytopenia in pregnancy
  • Mild: Counts > 70,000
  • Not associated with other abnormalities, and no symptoms
  • Usually develops in third trimester

Fischer, Conrad (2012-09-22). Master the Boards: USMLE Step 3 (Kaplan Medical Usmle Master the Boards Step 3) (Kindle Locations 12974-12995). Kaplan Publishing. Kindle Edition.

93
Q

HELLP syndrome occurs in 5– 10 percent of preeclamptic patients. It typically presents in the third trimester but may occur in the postpartum period,commonly presenting 2 days after delivery. Risk factors differ from preeclampsia, since HELLP syndrome is more common in whites, multigravids, and women of older maternal age.

Outline the treatment

Fischer, Conrad (2012-09-22). Master the Boards: USMLE Step 3 (Kaplan Medical Usmle Master the Boards Step 3) (Kindle Locations 12996-13007). Kaplan Publishing. Kindle Edition.

A
  • Schedule immediate delivery at any gestational age
  • Give IV corticosteroids (dexamethasone) when platelets < 100,000/mm3 both antepartum and postpartum; continue until platelet count is > 100,000/mm3 and liver function normalizes
  • Give platelet transfusion if platelet count < 20,000/mm3 or platelet count < 50,000/mm3 if cesarean section will be performed
  • IV MgSO4 for seizure prophylaxis, even if BP is normal
  • Steroids may also need to be considered for assistance with fetal lung maturation if prior to 36 weeks

Fischer, Conrad (2012-09-22). Master the Boards: USMLE Step 3 (Kaplan Medical Usmle Master the Boards Step 3) (Kindle Locations 12996-13007). Kaplan Publishing. Kindle Edition.

94
Q

List the complications of HELLP syndrome

A
  • DIC
  • Abruptio placenta
  • Fetal demise
  • Ascites
  • Hepatic rupture

Fischer, Conrad (2012-09-22). Master the Boards: USMLE Step 3 (Kaplan Medical Usmle Master the Boards Step 3) (Kindle Locations 12996-13007). Kaplan Publishing. Kindle Edition.

95
Q
  1. True or False: Heart disorders account for about 10 percent of maternal obstetric deaths
  2. True or False: Women with high-risk disorders (e.g., pulmonary hypertension, Eisenmenger syndrome, severe valvular disorders, prior postpartum cardiomyopathy) should be advised to become pregnant
  3. Cardiovascular changes in pregnancy (30– 50 percent ↑ cardiac output [CO]) may unmask or worsen underlying cardiac conditions. When are these changes maximal?
  4. Most dangerous seizure medication
  5. What seizure medication is/are proven safe in pregnancy

Fischer, Conrad (2012-09-22). Master the Boards: USMLE Step 3 (Kaplan Medical Usmle Master the Boards Step 3) (Kindle Locations 13009-13018). Kaplan Publishing. Kindle Edition.

A
  1. True
  2. False. They should be advised not to become pregnant due to risk of sudden death
  3. These changes are maximal at 28 and 34 weeks’ gestation
  4. Valproate
  5. None

Fischer, Conrad (2012-09-22). Master the Boards: USMLE Step 3 (Kaplan Medical Usmle Master the Boards Step 3) (Kindle Locations 13009-13018). Kaplan Publishing. Kindle Edition.

96
Q

Peripartum Cardiomyopathy

Heart failure with no identifiable cause can develop between the last month of pregnancy to 5 months postpartum.

List the risk factors

What is the 5 year mortality rate?

A

Risk factors

  • Multiparity
  • Age ≥ 30
  • Multiple gestations (i.e., twins or triplets, etc.), and preeclampsia

The 5-year mortality rate is 50 percent.

Fischer, Conrad (2012-09-22). Master the Boards: USMLE Step 3 (Kaplan Medical Usmle Master the Boards Step 3) (Kindle Locations 13009-13018). Kaplan Publishing. Kindle Edition.

97
Q
  1. True or False: Risk of maternal or fetal death is associated with class III or IV heart failure
  2. Medications NOT to use
  3. Medications to continue using
  4. True or False: Digoxin may be used in pregnancy to improve symptoms, but it does not improve outcome
A
  1. True
  2. ACE inhibitor or aldosterone antagonist
  3. Loop diuretics, nitrates, and ß-blockers
  4. True

Fischer, Conrad (2012-09-22). Master the Boards: USMLE Step 3 (Kaplan Medical Usmle Master the Boards Step 3) (Kindle Locations 13020-13030). Kaplan Publishing. Kindle Edition.

98
Q

True or False:

  1. Arrhythmias: Continue rate control as with nonpregnant patients
  2. Arrhythmias: Do not give amiodarone or warfarin
  3. Endocarditis Prophylaxis: Indications are the same as for nonpregnant patients
  4. Endocarditis Prophylaxis: Give daily prophylaxis in patients with rheumatic heart disease
  5. Endocarditis Prophylaxis: Do not give prophylactic antibiotics during uncomplicated vaginal/ cesarean delivery in patients with valvular disease or prosthetic valves
  6. Valvular Disease: Regurgitant lesions are well tolerated and do not require therapy
  7. Valvular Disease: Stenotic lesions have an increased risk of maternal/ fetal morbidity and mortality
  8. Valvular Disease: Mitral stenosis has an increased risk of pulmonary edema and atrial fibrillation

Fischer, Conrad (2012-09-22). Master the Boards: USMLE Step 3 (Kaplan Medical Usmle Master the Boards Step 3) (Kindle Locations 13020-13030). Kaplan Publishing. Kindle Edition.

A
  1. True
  2. True
  3. True
  4. True
  5. True
  6. True
  7. True
  8. True
99
Q

Pulmonary embolus is the leading cause of maternal death in the United States. Fifty percent of pregnant women who develop thromboemboli have an underlying thrombophilic disorder.

indications for anticoagulation in pregnancy?

A
  • DVT or PE in pregnancy
  • Atrial fibrillation (AF) with underlying heart disease, but not atrial fibrillation alone
  • Antiphospholipid syndrome
  • Severe heart failure (ejection fraction [EF]) < 30 percent)
  • Eisenmenger syndrome

Fischer, Conrad (2012-09-22). Master the Boards: USMLE Step 3 (Kaplan Medical Usmle Master the Boards Step 3) (Kindle Locations 13030-13051). Kaplan Publishing. Kindle Edition.

100
Q
  1. Anticoagulant of choice in pregnancy
  2. What are the advantages to answer to (1)
  3. Why is warfarin contraindicated in preganncy
A
  1. LMWH.
  2. LMWH does not cross the placenta (warfarin does cross the placenta and is contraindicated in pregnancy) and isn’t associated with osteopenia (unfractionated heparin causes osteopenia)
  3. Because it causes fetal abnormalities and even death. Patients with a history of DVT or PE in a previous pregnancy or a history of underlying thrombophilic condition should receive prophylactic LMWH throughout pregnancy,

Fischer, Conrad (2012-09-22). Master the Boards: USMLE Step 3 (Kaplan Medical Usmle Master the Boards Step 3) (Kindle Locations 13030-13051). Kaplan Publishing. Kindle Edition.

101
Q

Management of patients with a history of DVT or PE in a previous pregnancy or a history of underlying thrombophilic

True or False: Thrombolytics are used in pulmonary embolism with the same indications as in the nonpregnant patient

A

Management:

  • Prophylactic LMWH throughout pregnancy
  • Unfractionated heparin during labor and delivery
  • Warfarin for 6 weeks postpartum

True

Fischer, Conrad (2012-09-22). Master the Boards: USMLE Step 3 (Kaplan Medical Usmle Master the Boards Step 3) (Kindle Locations 13051-13067). Kaplan Publishing. Kindle Edition.

102
Q

List the most common underlying thrombophilias

A
  • Factor V Leiden mutation
  • Prothrombin gene mutation
  • Antiphospholipid syndrome
  • Hyperhomocysteinemia (MTHFR)
  • Antithrombin III deficiency

Fischer, Conrad (2012-09-22). Master the Boards: USMLE Step 3 (Kaplan Medical Usmle Master the Boards Step 3) (Kindle Locations 13051-13067). Kaplan Publishing. Kindle Edition.

103
Q
  1. Effect of hyperthyroidism on the fetus
  2. Effect of hypothyroidism on the fetus
  3. True or False: Pregnancy does not change the symptoms of hypothyroidism or hyperthyroidism or the normal values and ranges of free serum thyroxine (T4) and thyroid-stimulating hormone (TSH)
  4. True or False: Hormone replacement should be continued in patients with hypothyroidism during pregnancy
  5. True or False: The dose of levothyroxine should be incresed by 50 - 60 percent when hypothyroid patients become pregnant
  6. True or False: Triiodothyronine or desiccated thyroid are alternative medications in hypothyroidism
  7. True or False: Beta blockers are the drug of choice for symptomatic hyperthyroidism
  8. True or False: Radioactive iodine is never given in pregnancy
A
  1. Fetal growth restriction and stillbirth
  2. Intellectual deficits in offspring and miscarriage
  3. True
  4. True
  5. False. Increment should be by 25-30%
  6. False
  7. True
  8. True

Fischer, Conrad (2012-09-22). Master the Boards: USMLE Step 3 (Kaplan Medical Usmle Master the Boards Step 3) (Kindle Locations 13051-13067). Kaplan Publishing. Kindle Edition.

104
Q
  1. Drug of choice for Graves disease in pregnancy
  2. True or False: PTU crosses the placenta and may cause goiter and hypothyroidism in the fetus
  3. True or False: Congenital Graves’ disease in the fetus may be masked until 7 to 10 days after birth, when the drug’s effect subsides
  4. True or False: Maternal thyroid-stimulating immunoglobulins (Igs) and thyroid-blocking Igs can cross the placenta
  5. Effects of (4) on the fetus if any
A
  1. Propylthiouracil (PTU). (methimazole is the second line therapy).
  2. True
  3. True
  4. True
  5. Fetal tachycardia, growth restriction, and goiter

Fischer, Conrad (2012-09-22). Master the Boards: USMLE Step 3 (Kaplan Medical Usmle Master the Boards Step 3) (Kindle Locations 13067-13080). Kaplan Publishing. Kindle Edition.

105
Q

Diabetes in Pregnancy

  1. What are the target values for FBS and serum glucose 1 hour after meal
  2. Initial management of gestational diabetes (GD)
  3. Management of GD if (2) fails
  4. True or False: Insulin requirements increase throughout the course of the pregnancy including the immediate post-partum period
  5. Avoid oral hypoglycemics while breastfeeding, as they can cause hypoglycemia in neonates
A
  1. FBS < 90 mg/ dL and < 120 mg/ dL for 1 hour after a meal
  2. Diet and light exercise
  3. Insulin
  4. False. Insulin requirements increase throughout the course of the pregnancy but decrease as soon as the placenta is delivered
  5. True

Fischer, Conrad (2012-09-22). Master the Boards: USMLE Step 3 (Kaplan Medical Usmle Master the Boards Step 3) (Kindle Locations 13067-13080). Kaplan Publishing. Kindle Edition.

106
Q

List the routine monitoring in diabetic patients

A
  • HbA1c in each trimester
  • Triple-marker screen at 16– 18 weeks to assess for neural tube defects (NTD) Monthly sonograms to assess fetal macrosomia or IUGR
  • Monthly biophysical profiles
  • Start weekly nonstress test (NST) and amniotic fluid index (AFI) at 32 weeks if taking insulin, macrosomia, previous stillbirth, or hypertension
  • Start NSTs and AFIs at 26 weeks if small vessel disease is present or there is poor glycemic control.
  • For gestational diabetes mellitus (GDM) patients, order a 2-hour 75 g OGTT 6– 12 weeks postpartum to determine if diabetes has resolved. – Thirty-five percent of women with GDM will develop overt diabetes within 5 to 10 years after delivery.

Fischer, Conrad (2012-09-22). Master the Boards: USMLE Step 3 (Kaplan Medical Usmle Master the Boards Step 3) (Kindle Locations 13080-13095). Kaplan Publishing. Kindle Edition.

107
Q

True or False:

  1. Caudal regression syndrome is a common congenital abnormality associated with overt DM
  2. Congenital malformations (especially NTDs) are strongly associated with HbA1c > 8.5 in the first trimester
  3. GDM is not associated with congenital anomalies, since hyperglycemia is not present in the first half of pregnancy.
A
  1. True
  2. True
  3. True

Fischer, Conrad (2012-09-22). Master the Boards: USMLE Step 3 (Kaplan Medical Usmle Master the Boards Step 3) (Kindle Locations 13096-13110). Kaplan Publishing. Kindle Edition.

108
Q

Outline the management of labor in a diabetic patient

A
  • Larget delivery gestational age is 40 weeks because of delayed fetal maturity
  • Induce labor at 39– 40 weeks if < 4,500 g or earlier if there is poor glycemic control. A lecithin/ sphingomyelin (L/ S) ratio of 2.5 and presence of phosphatidyl glycerol ensures fetal lung maturity
  • Schedule cesarean section if > 4,500 g because of the risk of shoulder dystocia
  • Maintain maternal blood glucose levels between 80 and 100 mg/ dL using 5 percent dextrose in water and an insulin drip.
  • Turn off any insulin infusion after delivery, because insulin resistance decreases with rapidly falling levels of hPL after delivery of the placenta. Maintain blood glucose levels with a sliding scale

Fischer, Conrad (2012-09-22). Master the Boards: USMLE Step 3 (Kaplan Medical Usmle Master the Boards Step 3) (Kindle Locations 13096-13110). Kaplan Publishing. Kindle Edition.

109
Q

Complications of IODM

A
  • Hypoglycemia (maternal hyperinsulinemia)
  • Hypocalcemia (PTH synthesis failure)
  • Polycythemia (hypoxia)
  • Hyperbilirubinemia (excessive neonatal RBCs breakdown)
  • Respiratory distress syndrome (delayed surfactant production)
110
Q

Describe the symptoms, diagnosis, and Rx of intrahepatic cholestasis of pregnancy

A

Symptoms: Classic symptoms include intractable nocturnal pruritus on the palms and soles of the feet without skin findings

Diagnosis: 10- to 100-fold increase in serum bile acids

Treatment: Ursodeoxycholic acid is the treatment of choice. Symptoms may be relieved by antihistamines and cholestyramine.

Fischer, Conrad (2012-09-22). Master the Boards: USMLE Step 3 (Kaplan Medical Usmle Master the Boards Step 3) (Kindle Locations 13110-13127). Kaplan Publishing. Kindle Edition.

111
Q

How is the diagnosis of acute fatty liver made?

A
  • Elevated liver enzymes (e.g., ALT, AST, GGT)
  • Hyperbilirubinemia
  • DIC
  • Hypoglycemia
  • Increased serum ammonia

Hypoglycemia and increased serum ammonia are unique laboratory abnormalities.

Fischer, Conrad (2012-09-22). Master the Boards: USMLE Step 3 (Kaplan Medical Usmle Master the Boards Step 3) (Kindle Locations 13128-13160). Kaplan Publishing. Kindle Edition.

112
Q

Rx of acute fatty liver

A

Admit to the ICU for aggressive IV fluid treatment and prompt delivery

Fischer, Conrad (2012-09-22). Master the Boards: USMLE Step 3 (Kaplan Medical Usmle Master the Boards Step 3) (Kindle Locations 13128-13160). Kaplan Publishing. Kindle Edition.

113
Q

State the diagnosis, Rx and complication of the following:

  • Urine culture (+)
  • No urgency, frequency, or burning
  • No fever
A

Diagnosis: Asymptomatic bacteriuria

Rx: Outpatient PO antibiotics (Nitrofurantoin is drug of choice; alt: Cephalexin or amoxicillin)

Cx: 30% of cases develop acute pyelonephritis when untreated

Fischer, Conrad (2012-09-22). Master the Boards: USMLE Step 3 (Kaplan Medical Usmle Master the Boards Step 3) (Kindle Locations 13128-13160). Kaplan Publishing. Kindle Edition.

114
Q

State the diagnosis, Rx and complication of the following:

  • Urine culture (+)
  • Urgency, frequency, or burning
  • No fever
A

Diagnosis: Acute cystitis

Rx: Outpatient PO antibiotics (Nitrofurantoin is drug of choice; alt: Cephalexin or amoxicillin)

Cx: 30% of cases develop acute pyelonephritis when untreated

Fischer, Conrad (2012-09-22). Master the Boards: USMLE Step 3 (Kaplan Medical Usmle Master the Boards Step 3) (Kindle Locations 13128-13160). Kaplan Publishing. Kindle Edition.

115
Q

State the diagnosis, Rx and complication (Cx) of the following:

  • Urine culture (+)
  • Urgency, frequency, or burning
  • Fever and CVA tenderness
A

Diagnosis: Pyelonephritis

Rx:

  • Admit to hospital
  • IV hydration
  • IV cephalosporins or gentamicin
  • Tocolysis

Cx:

  • Preterm labor and delivery
  • Severe cases → sepsis, anemia, and pulmonary
116
Q

A 29-year-old primigravida presents to the emergency department at 19 weeks’ gestation with lower pelvic pressure without contractions. She reports increased clear vaginal mucus discharge. Fetal membranes are found to be bulging into the vagina, and the cervix cannot be palpated. Fetal feet can be felt through the membranes. What is the next step in management?

a. Abdominal cerclage
b. Prophylactic antibiotics
c. Tocolysis
d. Vaginal cerclage
e. Rule out chorioamnionitis

A

E. Although emergency cerclage (vaginal or abdominal) is indicated in women who present with cervical dilation in the absence of labor or abruption, it can only be performed when chorioamnionitis is first ruled out. Tocolysis is not appropriate in management of cervical insufficiency. Prophylactic antibiotics and tocolytics are not recommended

Fischer, Conrad (2012-09-22). Master the Boards: USMLE Step 3 (Kaplan Medical Usmle Master the Boards Step 3) (Kindle Locations 13272-13293). Kaplan Publishing. Kindle Edition.

117
Q

Indications for methotrexate in abortion

A
  • Pregnancy mass < 3.5 cm diameter
  • Absence of fetal heart motion
  • ß-hCG level < 6,000 mIU
  • No history of folic supplementation
118
Q

Risk factors for cervical insufficiency

A
  • Second-trimester abortion
  • Cervical laceration during delivery
  • Deep cervical conization
  • Diethylstilbestrol (DES) exposure

Fischer, Conrad (2012-09-22). Master the Boards: USMLE Step 3 (Kaplan Medical Usmle Master the Boards Step 3) (Kindle Locations 13272-13293). Kaplan Publishing. Kindle Edition.

119
Q

Rx of cervical insufficiency

A
  • Perform elective cerclage placement at 13– 16 weeks’ gestation for patients with ≥ 3 unexplained midtrimester pregnancy losses
  • Only perform urgent cerclage after first ruling out labor and chorioamnionitis
  • Perform cerclage removal at 36– 37 weeks, after fetal lung maturity

Fischer, Conrad (2012-09-22). Master the Boards: USMLE Step 3 (Kaplan Medical Usmle Master the Boards Step 3) (Kindle Locations 13293-13308). Kaplan Publishing. Kindle Edition.

120
Q

Intrauterine Growth Restriction (IUGR)

IUGR is the diagnosis when the estimated fetal weight (EFW) is < 5-10 percentile for gestational age or birthweight of < 2,500 g (5 lb, 8 oz).

Accurate, early pregnancy dating is essential for making the diagnosis. An early sonogram (< 20 weeks) is the next step in management if accurate dates are not known.

True or False: Never change the gestational age based on a late sonogram.

Fischer, Conrad (2012-09-22). Master the Boards: USMLE Step 3 (Kaplan Medical Usmle Master the Boards Step 3) (Kindle Locations 13293-13308). Kaplan Publishing. Kindle Edition.

A

True

Fischer, Conrad (2012-09-22). Master the Boards: USMLE Step 3 (Kaplan Medical Usmle Master the Boards Step 3) (Kindle Locations 13308-13354). Kaplan Publishing. Kindle Edition.

121
Q

List the causes of symmetric IUGR (Fetal causes)

↓ growth potential

A
  1. Aneuploidy
  2. Infection (e.g., TORCH)
  3. Structural anomalies
    • Congenital heart disease,
    • NTD
    • Ventral wall defects

Fischer, Conrad (2012-09-22). Master the Boards: USMLE Step 3 (Kaplan Medical Usmle Master the Boards Step 3) (Kindle Locations 13308-13354). Kaplan Publishing. Kindle Edition.

122
Q

List the causes of symmetric IUGR (Maternal causes)

↓ placental perfusion

A
  1. Hypertension
  2. Small vessel disease (e.g., SLE)
  3. Malnutrition
  4. Tobacco, alcohol, street drugs

Fischer, Conrad (2012-09-22). Master the Boards: USMLE Step 3 (Kaplan Medical Usmle Master the Boards Step 3) (Kindle Locations 13308-13354). Kaplan Publishing. Kindle Edition.

123
Q

List the causes of symmetric IUGR (Placental causes)

↓ placental perfusion

A
  • Infarction
  • Abruption
  • Twin-twin transfusion
  • Velamentous cord insertion

Fischer, Conrad (2012-09-22). Master the Boards: USMLE Step 3 (Kaplan Medical Usmle Master the Boards Step 3) (Kindle Locations 13308-13354). Kaplan Publishing. Kindle Edition.

124
Q

Ultrasound findings in IUGR

  1. Fetal causes
  2. Maternal and placental causes

Fischer, Conrad (2012-09-22). Master the Boards: USMLE Step 3 (Kaplan Medical Usmle Master the Boards Step 3) (Kindle Locations 13308-13354). Kaplan Publishing. Kindle Edition.

A
  1. ↓ in all measurements
  2. ↓ abdomen measurements; normal head measurements

Fischer, Conrad (2012-09-22). Master the Boards: USMLE Step 3 (Kaplan Medical Usmle Master the Boards Step 3) (Kindle Locations 13308-13354). Kaplan Publishing. Kindle Edition.

125
Q

List the workup for fetal causes of IUGR

Fischer, Conrad (2012-09-22). Master the Boards: USMLE Step 3 (Kaplan Medical Usmle Master the Boards Step 3) (Kindle Locations 13308-13354). Kaplan Publishing. Kindle Edition.

A
  • Detailed sonogram
  • Karyotype
  • Screen for fetal infections

Fischer, Conrad (2012-09-22). Master the Boards: USMLE Step 3 (Kaplan Medical Usmle Master the Boards Step 3) (Kindle Locations 13308-13354). Kaplan Publishing. Kindle Edition.

126
Q

List the workup for maternal and placenta causes of IUGR

Fischer, Conrad (2012-09-22). Master the Boards: USMLE Step 3 (Kaplan Medical Usmle Master the Boards Step 3) (Kindle Locations 13308-13354). Kaplan Publishing. Kindle Edition.

A
  1. Serial sonograms
  2. Nonstress test
  3. Amniotic fluid index (AFI)
  4. Biophysical profile
  5. Umbilical artery Doppler

AFI is often decreased, especially with severe uteroplacental insufficiency

Fischer, Conrad (2012-09-22). Master the Boards: USMLE Step 3 (Kaplan Medical Usmle Master the Boards Step 3) (Kindle Locations 13308-13354). Kaplan Publishing. Kindle Edition.

127
Q

Macrosomia is indicated by a fetus with estimated fetal weight (EFW) > 90– 95 percentile for gestational age or birth weight of 4,000– 4,500 g

List the risk factors for macrosomia

A
  • GDM
  • Overt diabetes
  • Prolonged gestation
  • Obesity
  • ↑ ↑ in pregnancy weight gain
  • Multiparity
  • Male fetus

Fischer, Conrad (2012-09-22). Master the Boards: USMLE Step 3 (Kaplan Medical Usmle Master the Boards Step 3) (Kindle Locations 13355-13378). Kaplan Publishing. Kindle Edition.

128
Q

List the complications and management of fetal macrosomia

A

Complications include:

  1. Maternal:
    • Injury during birth
    • Postpartum hemorrhage
    • Emergency cesarean section
  2. Fetus:
    • Shoulder dystocia
    • Birth injury
    • Asphyxia
  3. Neonate:
    • Hypoglycemia
    • Erb palsy

Management: Elective cesarean (if EFW > 4,500 g in diabetic mother or > 5,000 g in nondiabetic mother)

Fischer, Conrad (2012-09-22). Master the Boards: USMLE Step 3 (Kaplan Medical Usmle Master the Boards Step 3) (Kindle Locations 13355-13378). Kaplan Publishing. Kindle Edition.

129
Q

Premature Rupture of Membranes (PROM):

This is rupture of the fetal membranes before the onset of labor.

What is the most common risk factor for PROM?

A

Ascending infection from the lower genital tract

Fischer, Conrad (2012-09-22). Master the Boards: USMLE Step 3 (Kaplan Medical Usmle Master the Boards Step 3) (Kindle Locations 13401-13411). Kaplan Publishing. Kindle Edition.

130
Q

How is the diagnosis of PROM made?

A
  1. Sterile speculum examination reveals the following:
    • There is posterior fornix pooling of clear amniotic fluid (AF)
    • Fluid is nitrazine positive
    • Fluid is ferning positive
  2. Ultrasound: Oligohydramnios
131
Q

Criteria for clinically diagnosis of chorioamnionitis

A
  • Maternal fever and uterine tenderness
  • Confirmed PROM
  • Absence of a URI or UTI

Fischer, Conrad (2012-09-22). Master the Boards: USMLE Step 3 (Kaplan Medical Usmle Master the Boards Step 3) (Kindle Locations 13425-13428). Kaplan Publishing. Kindle Edition.

132
Q

Outline the management of PROM under the follwing circumstances

  1. Presence of uterine contractions
  2. Presence of chorioamnionitis
  3. Absence of infection before viability (< 24 weeks)
  4. Absence of infection with preterm viability (24-33 weeks)
  5. Absence of infection at term (34 weeks)
A
  1. If uterine contractions are present, do not give tocolysis
  2. If chorioamnionitis is present
    • Get cervical cultures
    • Start IV antibiotics
    • Schedule delivery
  3. Manage patient with bed rest at home
  4. Hospitalize
    • Give IM betamethasone if < 32 weeks
    • Obtain cervical cultures
    • Begin prophylactic ampicillin and erythromycin for 7 days
  5. At term (> 34 weeks): Initiate delivery.

Fischer, Conrad (2012-09-22). Master the Boards: USMLE Step 3 (Kaplan Medical Usmle Master the Boards Step 3) (Kindle Locations 13425-13428).

133
Q

Obstetric ultrasound

  1. What does the crown–rump length determines?
  2. When is the crown-rump length done
  3. What are the indications for high resolution ultrasonography
A
  1. Determines gestational age
  2. At 10 – 13 weeks
  3. Abnormal maternal serum markers or a family history of congenital malformations

Fischer, Conrad (2012-09-22). Master the Boards: USMLE Step 3 (Kaplan Medical Usmle Master the Boards Step 3) (Kindle Locations 13355-13378). Kaplan Publishing. Kindle Edition.

134
Q

Chorionic villous sampling (CVS)

  1. When is CVS perfomed?
  2. Why is it performed
  3. What is the pregnancy loss rate
A
  1. At 12 – 14 weeks
  2. For karyotyping
  3. 0.7%

Fischer, Conrad (2012-09-22). Master the Boards: USMLE Step 3 (Kaplan Medical Usmle Master the Boards Step 3) (Kindle Locations 13355-13378). Kaplan Publishing. Kindle Edition.

135
Q

Amniocentesis

When is it perfomed?
Why is it performed
What is the pregnancy loss rate

A
  1. After 15 weeks with ultrasound guidance
  2. For karyotyping. AF AFP and acetylcholinesterase from amniotic fluid screens for neural tube defect
  3. 0.5%

Fischer, Conrad (2012-09-22). Master the Boards: USMLE Step 3 (Kaplan Medical Usmle Master the Boards Step 3) (Kindle Locations 13355-13378). Kaplan Publishing. Kindle Edition.

136
Q

Percutaneous umbilical blood sample

  1. How is it performed?
  2. When is it perfomed?
  3. Why is it performed
  4. What is the pregnancy loss rate
A
  1. Involves ultrasound guided aspiration of fetal blood from the umbilical vein
  2. After 20 weeks’ gestation
  3. Is both diagnostic (e.g., blood gases, karyotype, IgG and IgM antibodies) and therapeutic (e.g., intrauterine transfusion with fetal anemia)
  4. 1– 2 percent

Fischer, Conrad (2012-09-22). Master the Boards: USMLE Step 3 (Kaplan Medical Usmle Master the Boards Step 3) (Kindle Locations 13379-13405). Kaplan Publishing. Kindle Edition.

137
Q

Fetoscopy

  1. How is it performed?
  2. When is it perfomed?
  3. Why is it performed?
  4. What is the pregnancy loss rate?
A
  1. Transabdominal fiberoptic scope under regional or general anesthesia
  2. After 20 weeks
  3. Indications include intrauterine surgery or fetal skin biopsy (ichthyosis)
  4. 2– 5 percent

Fischer, Conrad (2012-09-22). Master the Boards: USMLE Step 3 (Kaplan Medical Usmle Master the Boards Step 3) (Kindle Locations 13379-13405). Kaplan Publishing. Kindle Edition.

138
Q

Cervical cerclage

  1. How is it performed?
  2. When is it perfomed?
  3. Why is it performed?
A
  1. Suture encircles the cervix to prevent the cervical canal from dilating
  2. Between 14 and 24 weeks
  3. Indicated electively or emergently in cervical insufficiency

Fischer, Conrad (2012-09-22). Master the Boards: USMLE Step 3 (Kaplan Medical Usmle Master the Boards Step 3) (Kindle Locations 13379-13405). Kaplan Publishing. Kindle Edition.

139
Q

Describe the features of an adequate uterine contraction (UC)

A
  • Occurs every 2– 3 minutes
  • Lasts 45– 60 seconds
  • Has 50 mm Hg intensity

Fischer, Conrad (2012-09-22). Master the Boards: USMLE Step 3 (Kaplan Medical Usmle Master the Boards Step 3) (Kindle Locations 13417-13449). Kaplan Publishing. Kindle Edition.

140
Q

Stage 1— Latent phase effacement

  1. Define it
  2. Duration
  3. Features of prolonged latent phase
  4. Most common cause of (3)
  5. Management (3)
A
  1. Begins: Onset of regular UC; Ends: Acceleration of cervical dilatation
  2. < 20 hours (primipara); < 14 hours (multipara)
  3. Prolonged latent phase:
    • Cervix dilated < 3 cm
    • No cervicl change in 20 h (primipara)/14 h (multipara)
  4. Analgesia
  5. Rest and sedation

Fischer, Conrad (2012-09-22). Master the Boards: USMLE Step 3 (Kaplan Medical Usmle Master the Boards Step 3) (Kindle Locations 13417-13449). Kaplan Publishing. Kindle Edition.

141
Q

Stage 1— Active phase dilation

This stage prepares cervix for dilation

  1. Define it
  2. Duration
  3. Features of active phase prolongation
  4. Features of active phase arrest
  5. Common cause of (3/4)
  6. Management (3/4)
A
  1. Begins: Acceleration of cervical dilation Ends: 10 cm (complete) Rapid cervical dilation
  2. > 1.2 cm/ hour (primipara) or > 1.5 cm/ hour (multipara)
  3. Active phase prolongation:
    • Cervix dilated ≥ 3cm
    • Cervical dilation of < 1.2 cm/ h (primipara) / < 1.5 cm/ h (multipara)
  4. Active phase arrest:
    • Cervix dilated ≥ 3cm
    • No cervical change in ≥ 2h
  5. Causes:
    • Abnormalities with passenger (fetal size or abnormal presentation)
    • Abnormalities with the pelvis or power (dysfunctional contractions)
  6. Management:
    • Hypotonic contractions → IV oxytocin
    • Hypertonic contractions → morphine sedation
    • Adequate contractions → emergency cesarean section

Fischer, Conrad (2012-09-22). Master the Boards: USMLE Step 3 (Kaplan Medical Usmle Master the Boards Step 3) (Kindle Locations 13442-13502). Kaplan Publishing. Kindle Edition.

142
Q

Stage 2— Descent

  1. Define it
  2. Duration
  3. Features of second stage arrest
  4. Common causes of (3)
  5. Management (3)
A
  1. Begins: 10 cm (complete) Ends: delivery of baby Descent of the fetus
  2. < 2 hours (primipara) or < 1 hour (multipara). Add 1 hour if epidural
  3. Second-stage arrest
    • Failure to deliver within 2 hours (primipara) or 1 hour (multipara)
    • Add additional 1 hour if epidural
  4. Cause: Abnormalities with passenger, pelvis, or power
  5. Management:
    • Fetal head is not engaged → emergency cesarean
    • Fetal head is engaged → trial of obstetric forceps or vacuum extraction

Fischer, Conrad (2012-09-22). Master the Boards: USMLE Step 3 (Kaplan Medical Usmle Master the Boards Step 3) (Kindle Locations 13442-13502). Kaplan Publishing. Kindle Edition.

143
Q

Stage 3— Expulsion

  1. Define it
  2. Duration
  3. Features of second stage arrest
  4. Common causes of (3)
  5. Management (3)
A
  1. Begins: delivery of baby Ends: delivery of placenta Delivery of placenta
  2. < 30 minutes
  3. Prolonged third stage: Failure to deliver placenta within 30 minutes
  4. Cause: Consider placenta accreta/ increta/ percreta
  5. Management:
    • IV oxytocin
    • If oxytocin fails, attempt manual removal
    • Hysterectomy may be needed

Fischer, Conrad (2012-09-22). Master the Boards: USMLE Step 3 (Kaplan Medical Usmle Master the Boards Step 3) (Kindle Locations 13442-13502). Kaplan Publishing. Kindle Edition.

144
Q

Umbilical Cord Prolapse (UCP)

An obstetric emergency because a compressed cord has jeopardized fetal oxygenation, cord prolapse most often occurs with rupture of membranes before the head is engaged in breech of transverse lie. A fetal heart rate (FHR) rate that suggests hypoxemia (e.g., severe bradycardia, severe variable accelerations) may be the only clue

Outline the management of UCP

A
  • Never attempt to replace the cord
  • Place the patient in knee-chest position, elevate the presenting part, and give terbutaline to decrease force of contractions
  • Perform immediate cesarean delivery

Fischer, Conrad (2012-09-22). Master the Boards: USMLE Step 3 (Kaplan Medical Usmle Master the Boards Step 3) (Kindle Locations 13503-13518). Kaplan Publishing. Kindle Edition.

145
Q

Baseline heart rate is the mean FHR during a 10-minute segment of time, excluding periodic changes.

List the common causes of changes in fetal heart rate

A
  • Uterine hyperstimulation (commonly caused by medications)
  • Fetal head compression
  • Umbilical cord compression
  • Placental insufficiency

Fischer, Conrad (2012-09-22). Master the Boards: USMLE Step 3 (Kaplan Medical Usmle Master the Boards Step 3) (Kindle Locations 13503-13518). Kaplan Publishing. Kindle Edition.

146
Q

Define

  1. Normal baseline FHR
  2. Tachycardia
  3. Bradycardia
A
  1. 110– 160 beats/ minute
  2. > 160 beats/ minute is most commonly related to medications (β-agonist: terbutaline, ritodrine)
  3. < 110 beats/ minute is most commonly related to medications (β-blockers or local anesthetics)
    1. . Reassuring FHR tracing: • Baseline FHR 110– 160 beats/ minute • (+) Accelerations • (-) Decelerations • (+) Beat-to-beat variability Nonreassuring FHR tracing: • Baseline FHR shows tachycardia or bradycardia • (–) Accelerations • (+) Variable or late decelerations • (–) Beat-to-beat variability Periodic changes in heart rate include the following: Accelerations: Abrupt increases in FHR lasting < 2 minutes that are unrelated to contractions. They always occur in

Fischer, Conrad (2012-09-22). Master the Boards: USMLE Step 3 (Kaplan Medical Usmle Master the Boards Step 3) (Kindle Locations 13519-13536). Kaplan Publishing. Kindle Edition.

147
Q

Features of reassuring fetal heart tracing

A
  • Baseline FHR 110 – 160 beats/ minute
  • (+) Accelerations
  • (-) Decelerations
  • (+) Beat-to-beat variability
  • Nonreassuring FHR tracing: • Baseline FHR shows tachycardia or bradycardia • (–) Accelerations • (+) Variable or late decelerations • (–) Beat-to-beat variability Periodic changes in heart rate include the following: Accelerations: Abrupt increases in FHR lasting < 2 minutes that are unrelated to contractions. They always occur in

Fischer, Conrad (2012-09-22). Master the Boards: USMLE Step 3 (Kaplan Medical Usmle Master the Boards Step 3) (Kindle Locations 13519-13536). Kaplan Publishing. Kindle Edition.

148
Q

Features of non-reassuring fetal heart tracing

A
  • Baseline FHR shows tachycardia or bradycardia
  • (–) Accelerations
  • (+) Variable or late decelerations
  • (–) Beat-to-beat variability

Fischer, Conrad (2012-09-22). Master the Boards: USMLE Step 3 (Kaplan Medical Usmle Master the Boards Step 3) (Kindle Locations 13519-13536). Kaplan Publishing. Kindle Edition.

149
Q

Spot diagnosis

What does this indicate?

A

Early decelerations.

Indicates head compression

Fischer, Conrad (2012-09-22). Master the Boards: USMLE Step 3 (Kaplan Medical Usmle Master the Boards Step 3) (Kindle Locations 13519-13536). Kaplan Publishing. Kindle Edition.

150
Q

Spot diagnosis

What does this indicate?

A

Variable decelerations

Indicates umbilical cord compression

Fischer, Conrad (2012-09-22). Master the Boards: USMLE Step 3 (Kaplan Medical Usmle Master the Boards Step 3) (Kindle Locations 13536-13542). Kaplan Publishing. Kindle Edition.

151
Q

Spot diagnosis

What does this indicate?

A

Late decelerations

Uteroplacental insufficiency

Fischer, Conrad (2012-09-22). Master the Boards: USMLE Step 3 (Kaplan Medical Usmle Master the Boards Step 3) (Kindle Locations 13536-13542). Kaplan Publishing. Kindle Edition.

152
Q

List and describe the periodic changes in fetal heart rate

A

Accelerations: Abrupt increases in FHR lasting < 2 minutes that are unrelated to contractions. They always occur in response to fetal movements and are always reassuring.

Early decelerations: Gradual decreases in FHR beginning and ending simultaneously with contractions. They occur in response to fetal head compression.

Variable decelerations: Abrupt decreases in FHR that are unrelated to contractions. These are related to umbilical cord compression. Severe variables are nonreassuring and indicate fetal acidosis.

Late decelerations are gradual decreases in FHR and delayed in relation to contractions. These are related to uteroplacental insufficiency. All late decelerations are nonreassuring and indicate fetal acidosis.

Variability: Beat-to-beat fetal heart rate normally has variability. Normal variability is 6– 25 beats/ minute. Absence of variability is a nonreassuring pattern.

Fischer, Conrad (2012-09-22). Master the Boards: USMLE Step 3 (Kaplan Medical Usmle Master the Boards Step 3) (Kindle Locations 13543-13548). Kaplan Publishing. Kindle Edition.

153
Q

List the non-reassuring changes in fetal heart rate patterns

A
  • Variable decelerations
  • Late decelerations
  • Absence of beat-to-beat variability

Fischer, Conrad (2012-09-22). Master the Boards: USMLE Step 3 (Kaplan Medical Usmle Master the Boards Step 3) (Kindle Locations 13543-13548). Kaplan Publishing. Kindle Edition.

154
Q

A 31-year-old primigravida at term is in the maternity unit in active labor. She is 6 cm dilated, 100 percent effaced 0 station, with the fetus in cephalad position. IV oxytocin is being administered because of arrest of cervical dilation at 6 cm. Fetal membranes are intact. The nurse informs you that the external fetal monitor tracing now shows the fetal heart rate baseline at 175/ minute with minimal variability and repetitive late decelerations. There is no vaginal bleeding. What is the most appropriate next step in management?

a. Change maternal position
b. Discontinue oxytocin
c. Immediate cesarean section
d. Perform obstetric ultrasound
e. Obtain fetal scalp pH

A

B. Medications are a common cause of baseline fetal tachycardia or bradycardia

Fischer, Conrad (2012-09-22). Master the Boards: USMLE Step 3 (Kaplan Medical Usmle Master the Boards Step 3) (Kindle Locations 13549-13573). Kaplan Publishing. Kindle Edition.

155
Q

Outline the stepwise approach to non-reassuring fetal tracings

A
  1. Examine the electronic fetal monitoring (EFM) strip: Look for non-reassuring patterns
  2. Identify nonhypoxic causes that can explain the abnormal findings. (Most common are medications, particularly β-agonists or β-blockers.)
  3. Begin intrauterine resuscitation as follows:
    • Discontinue medications (e.g., oxytocin)
    • Give IV normal saline bolus
    • Provide high-flow oxygen
    • Change patient’s position (left lateral)
    • Vaginal exam to rule out prolapsed cord
    • Perform scalp stimulation to observe for accelerations (reassuring)
  4. Prepare for delivery if the EFM tracing does not normalize
  5. If the EFM is unequivocal, obtain fetal scalp pH (requires dilated cervix and ruptured membranes). Normal fetal pH > 7.20

Fischer, Conrad (2012-09-22). Master the Boards: USMLE Step 3 (Kaplan Medical Usmle Master the Boards Step 3) (Kindle Locations 13549-13573). Kaplan Publishing. Kindle Edition.

156
Q

Indications for Forceps- or Vacuum-Assisted Delivery

A
  • Prolonged second stage (most common indication)
  • Nonreassuring EFM strip in absence of contraindications
  • To avoid maternal pushing when mother has cardiac and/ or pulmonary conditions that would increase her risk

Fischer, Conrad (2012-09-22). Master the Boards: USMLE Step 3 (Kaplan Medical Usmle Master the Boards Step 3) (Kindle Locations 13574-13587). Kaplan Publishing. Kindle Edition.

157
Q

Contraindications to Forceps- or Vacuum-Assisted Delivery

A
  • Mother has small pelvis
  • Cervix is not fully dilated
  • Membranes have not ruptured
  • Fetal head is not engaged
  • Orientation of the head is not certain

Fischer, Conrad (2012-09-22). Master the Boards: USMLE Step 3 (Kaplan Medical Usmle Master the Boards Step 3) (Kindle Locations 13574-13587). Kaplan Publishing. Kindle Edition.

158
Q

The cesarean section rate is approximately 33 percent in the United States (including both primary and repeat procedures).

Risks include the following: Increased risk of hemorrhage, infection, visceral injury (bladder, bowel, ureters), and DVTs.

Low segment transverse incision: This is the most common procedure. It can only be performed with longitudinal lie of the fetus.

Classic vertical incision: Can be performed with any fetal lie. Because of the increased risk of uterine rupture in subsequent pregnancies, cesarean must be initiated before labor begins.

Indications for CS

Fischer, Conrad (2012-09-22). Master the Boards: USMLE Step 3 (Kaplan Medical Usmle Master the Boards Step 3) (Kindle Locations 13574-13587). Kaplan Publishing. Kindle Edition.

A

Cephalopelvic disproportion (CPD): With failure of progression or arrest in labor

Fetal malpresentation: Most commonly preterm breech and nonfrank breech

Nonreassuring EFM strip

Placenta previa (unless placental edge > 2 cm from internal os)

Infection: Mother who is HIV-positive or has active vaginal herpes

Uterine scar: Prior myomectomy (fibroid) or prior classic incision c-section

Trial of vaginal birth after cesarean (VBAC) should be attempted in patients in the absence of c-section indications when the previous cesarean was a low segment uterine incision. The success rate is 80 percent.

Fischer, Conrad (2012-09-22). Master the Boards: USMLE Step 3 (Kaplan Medical Usmle Master the Boards Step 3) (Kindle Locations 13587-13600). Kaplan Publishing. Kindle Edition.

159
Q

External cephalic version (ECV)

  1. Indications
  2. Optimal time
  3. Success rate
A
  1. Transverse lie or breech presentation
  2. 37 weeks’ gestation
  3. 60– 70 percent

External cephalic version (ECV) is done to change a baby from breech or other non-cephalic presentation to the cephalic position. The physician pushes on the baby through the mother’s abdomen to attempt to roll the baby into position

Fischer, Conrad (2012-09-22). Master the Boards: USMLE Step 3 (Kaplan Medical Usmle Master the Boards Step 3) (Kindle Locations 13600-13616). Kaplan Publishing. Kindle Edition.

160
Q

List the causes of post-partum hemorrhage

A
  • Uterine atony (most common cause of excessive postpartum bleeding)
  • Lacerations
  • Retained placenta
  • DIC
  • Uterine inversion
  • Urinary retention

Fischer, Conrad (2012-09-22). Master the Boards: USMLE Step 3 (Kaplan Medical Usmle Master the Boards Step 3) (Kindle Locations 13600-13616). Kaplan Publishing. Kindle Edition.

161
Q

Causes of DIC in pregnancy/labor/delivery

Management of retained placenta

A

DIC (Suspect DIC when there is generalized oozing or bleeding from IV or laceration sites in the presence of a contracted uterus)

  • Abruptio placenta
  • Severe preeclampsia
  • Amniotic fluid embolism
  • Prolonged retention of a dead fetus

Management of retained placenta

  • Manual removal
  • Uterine curretage under ultrasound guidance
  • Hysterectomy may be indicated

Fischer, Conrad (2012-09-22). Master the Boards: USMLE Step 3 (Kaplan Medical Usmle Master the Boards Step 3) (Kindle Locations 13616-13628). Kaplan Publishing. Kindle Edition.

162
Q

Presentation and management of Uterine inversion

Rx of urinary retention

A

Uterine inversion:

  • Beefy-appearing bleeding mass in the vagina
  • Failure to palpate the uterus
  • Management involves uterine replacement, followed by IV oxytocin.

Urinary retention may occur with a hypotonic bladder.

  • If residual volume is > 250 mL, give bethanechol (Urecholine)
  • If this fails, urinary catheterize for no more than 2– 3 days

Fischer, Conrad (2012-09-22). Master the Boards: USMLE Step 3 (Kaplan Medical Usmle Master the Boards Step 3) (Kindle Locations 13616-13628). Kaplan Publishing. Kindle Edition.

163
Q

Special notes on Postpartum Contraception in:

Breastfeeding

Diaphrag and IUD placement

Combined estrogen-progestin formulations

Progestin contraception

A

Breastfeeding:

  • Contraceptive use may be deferred for 3 months in women who are breastfeeding because of temporary anovulation

Diaphragm and IUD placement:

  • Deferred until the 6- week postpartum visit

Combined estrogen-progestin formulations (e.g., pills, patch, vaginal ring):

  • Not started until 3 weeks postpartum to prevent hypercoagulable state and risk of DVT
  • They are not used in breastfeeding women because of diminished lactation.

Progestin contraception (e.g., mini-pill, Depo-Provera, Implanon):

  • Can safely be used during breastfeeding
  • They can be begun immediately after delivery.

Fischer, Conrad (2012-09-22). Master the Boards: USMLE Step 3 (Kaplan Medical Usmle Master the Boards Step 3) (Kindle Locations 13629-13678). Kaplan Publishing. Kindle Edition.

164
Q

Based on the post-partum days list the diagnosis, risk factors, clinical findings and management of the different causes of PP fever.

Day 0

Day 1

day 2-3

Day 4-5

Day 5-6

Day 7-21

A

See table for answers